Slot System
Featured Buckets
Featured Buckets Admin

Medications that scare me

Article Type
Changed
Wed, 05/17/2023 - 09:14

An 85-year-old woman is brought to the emergency department after a syncopal episode. Her caregivers report a similar episode 2 weeks ago, but she recovered so quickly they did not seek evaluation for her.

Paauw_Doug_SEATTLE_2019_web.jpg
Dr. Douglas S. Paauw

Medications: Omeprazole 20 mg, pravastatin 40 mg, citalopram 10 mg, albuterol, donepezil 10 mg, isosorbide mononitrate 60 mg, and calcium. On exam, blood pressure is 100/60 mm Hg, pulse 55. ECG indicates bradycardia with normal intervals. What drug most likely caused her syncope?

A. Citalopram

B. Pravastatin

C. Donepezil

D. Isosorbide

E. Calcium

This woman’s syncope is likely caused by donepezil. Citalopram can lengthen the QT interval, especially in elderly patients, but the normal intervals on ECG eliminate this possibility. Donepezil can cause bradycardia, which can contribute to syncope.

Hernandez and colleagues evaluated a cohort of veterans with dementia over an 8-year period.1 They found that there was a 1.4-fold increased risk of bradycardia in patients with dementia treated with an acetylcholine inhibitor (compared with that in patients who were not taking these medications) and that there was a dose-dependent increase in risk for patients on donepezil.

Park-Wyllie et al. found in a study of 1.4 million older adults a greater than twofold risk of hospitalization for bradycardia in patients treated with a cholinesterase inhibitor.2 Gill and colleagues performed a population-based cohort study of 19,803 elderly patients with dementia who were prescribed cholinesterase inhibitors, and compared them to age-matched controls.3 They found increased hospital visits for syncope in people receiving cholinesterase inhibitors (hazard ratio, 1.76; 95% confidence interval, 1.57-1.98). Other syncope-related events were also more common in people receiving cholinesterase inhibitors, compared with controls: hospital visits for bradycardia (HR, 1.69; 95% CI, 1.32-2.15), permanent pacemaker insertion (HR, 1.49; 95% CI, 1.12-2.00), and hip fracture (HR, 1.18; (95% CI, 1.04-1.34).

Nausea, vomiting, and weight loss are much more common than the rarer side effects of bradycardia and syncope. The frequency of gastroenterological side effects is up to 25%. Cholinesterase inhibitors have modest effects on cognitive function with a high number needed to treat (NNT) of 10, and an NNT as high as 100 for global function. The number needed to harm (NNH) is 4, when gastrointestinal symptoms are added in.4 Another important, problematic side effect of cholinesterase inhibitors is urinary incontinence. This often leads to patients receiving medications, to combat this side effect, that may worsen cognitive function.

Another commonly used medication that scares me in certain circumstances is trimethoprim-sulfamethoxazole. My main concern is when it is used in patients who are elderly, have chronic kidney disease, or are taking other medications that can cause hyperkalemia (ACEIs, ARBs, potassium-sparing diuretics including spironolactone). Hyperkalemia is a real concern in these patient populations. Trimethoprim reduces renal potassium excretion through the competitive inhibition of sodium channels in the distal nephron, in a manner similar to the potassium-sparing diuretic amiloride. Hospitalizations for hyperkalemia are more common in patients who take ACEIs and ARBs and are prescribed trimethoprim-sulfamethoxazole, compared with other antibiotics.5

Sudden cardiac death is also more common in patients who are taking ACEIs or ARBs and receive trimethoprim-sulfamethoxazole.6 Trimethoprim-sulfamethoxazole also has a powerful interaction with warfarin, both displacing warfarin from albumin and inhibiting its metabolism. It raises the INR (international normalized ratio) in warfarin-treated patients much greater than do other antibiotics.7
 

Pearls

  • Think carefully about the use of cholinesterase inhibitors because of the unfavorable NNH vs. NNT.
  • Use caution prescribing trimethoprim for patients who are elderly, especially if they are on an ACEI, an ARB, or spironolactone, and in patients with chronic kidney disease.

Dr. Paauw is professor of medicine in the division of general internal medicine at the University of Washington, Seattle, and he serves as third-year medical student clerkship director at the University of Washington. Contact Dr. Paauw at dpaauw@uw.edu.

References

1. Hernandez RK et al. J Am Geriatr Soc. 2009;57:1997-2003.

2. Park-Wyllie LY et al. PLoS Med. 2009;6:e1000157.

3. Gill SS et al. Arch Intern Med 2009;169:867-73.

4. Peters KR. J Am Geriatr Soc. 2013 Jul;61(7):1170-4.

5. Antoniou TN et al. Arch Intern Med. 2010;170(12):1045-9.

6. Fralick M et al. BMJ. 2014 Oct 30;349:g6196.

7. Glasheen JJ et al. J Gen Intern Med. 2005 Jul;20(7):653-6.

Publications
Topics
Sections

An 85-year-old woman is brought to the emergency department after a syncopal episode. Her caregivers report a similar episode 2 weeks ago, but she recovered so quickly they did not seek evaluation for her.

Paauw_Doug_SEATTLE_2019_web.jpg
Dr. Douglas S. Paauw

Medications: Omeprazole 20 mg, pravastatin 40 mg, citalopram 10 mg, albuterol, donepezil 10 mg, isosorbide mononitrate 60 mg, and calcium. On exam, blood pressure is 100/60 mm Hg, pulse 55. ECG indicates bradycardia with normal intervals. What drug most likely caused her syncope?

A. Citalopram

B. Pravastatin

C. Donepezil

D. Isosorbide

E. Calcium

This woman’s syncope is likely caused by donepezil. Citalopram can lengthen the QT interval, especially in elderly patients, but the normal intervals on ECG eliminate this possibility. Donepezil can cause bradycardia, which can contribute to syncope.

Hernandez and colleagues evaluated a cohort of veterans with dementia over an 8-year period.1 They found that there was a 1.4-fold increased risk of bradycardia in patients with dementia treated with an acetylcholine inhibitor (compared with that in patients who were not taking these medications) and that there was a dose-dependent increase in risk for patients on donepezil.

Park-Wyllie et al. found in a study of 1.4 million older adults a greater than twofold risk of hospitalization for bradycardia in patients treated with a cholinesterase inhibitor.2 Gill and colleagues performed a population-based cohort study of 19,803 elderly patients with dementia who were prescribed cholinesterase inhibitors, and compared them to age-matched controls.3 They found increased hospital visits for syncope in people receiving cholinesterase inhibitors (hazard ratio, 1.76; 95% confidence interval, 1.57-1.98). Other syncope-related events were also more common in people receiving cholinesterase inhibitors, compared with controls: hospital visits for bradycardia (HR, 1.69; 95% CI, 1.32-2.15), permanent pacemaker insertion (HR, 1.49; 95% CI, 1.12-2.00), and hip fracture (HR, 1.18; (95% CI, 1.04-1.34).

Nausea, vomiting, and weight loss are much more common than the rarer side effects of bradycardia and syncope. The frequency of gastroenterological side effects is up to 25%. Cholinesterase inhibitors have modest effects on cognitive function with a high number needed to treat (NNT) of 10, and an NNT as high as 100 for global function. The number needed to harm (NNH) is 4, when gastrointestinal symptoms are added in.4 Another important, problematic side effect of cholinesterase inhibitors is urinary incontinence. This often leads to patients receiving medications, to combat this side effect, that may worsen cognitive function.

Another commonly used medication that scares me in certain circumstances is trimethoprim-sulfamethoxazole. My main concern is when it is used in patients who are elderly, have chronic kidney disease, or are taking other medications that can cause hyperkalemia (ACEIs, ARBs, potassium-sparing diuretics including spironolactone). Hyperkalemia is a real concern in these patient populations. Trimethoprim reduces renal potassium excretion through the competitive inhibition of sodium channels in the distal nephron, in a manner similar to the potassium-sparing diuretic amiloride. Hospitalizations for hyperkalemia are more common in patients who take ACEIs and ARBs and are prescribed trimethoprim-sulfamethoxazole, compared with other antibiotics.5

Sudden cardiac death is also more common in patients who are taking ACEIs or ARBs and receive trimethoprim-sulfamethoxazole.6 Trimethoprim-sulfamethoxazole also has a powerful interaction with warfarin, both displacing warfarin from albumin and inhibiting its metabolism. It raises the INR (international normalized ratio) in warfarin-treated patients much greater than do other antibiotics.7
 

Pearls

  • Think carefully about the use of cholinesterase inhibitors because of the unfavorable NNH vs. NNT.
  • Use caution prescribing trimethoprim for patients who are elderly, especially if they are on an ACEI, an ARB, or spironolactone, and in patients with chronic kidney disease.

Dr. Paauw is professor of medicine in the division of general internal medicine at the University of Washington, Seattle, and he serves as third-year medical student clerkship director at the University of Washington. Contact Dr. Paauw at dpaauw@uw.edu.

References

1. Hernandez RK et al. J Am Geriatr Soc. 2009;57:1997-2003.

2. Park-Wyllie LY et al. PLoS Med. 2009;6:e1000157.

3. Gill SS et al. Arch Intern Med 2009;169:867-73.

4. Peters KR. J Am Geriatr Soc. 2013 Jul;61(7):1170-4.

5. Antoniou TN et al. Arch Intern Med. 2010;170(12):1045-9.

6. Fralick M et al. BMJ. 2014 Oct 30;349:g6196.

7. Glasheen JJ et al. J Gen Intern Med. 2005 Jul;20(7):653-6.

An 85-year-old woman is brought to the emergency department after a syncopal episode. Her caregivers report a similar episode 2 weeks ago, but she recovered so quickly they did not seek evaluation for her.

Paauw_Doug_SEATTLE_2019_web.jpg
Dr. Douglas S. Paauw

Medications: Omeprazole 20 mg, pravastatin 40 mg, citalopram 10 mg, albuterol, donepezil 10 mg, isosorbide mononitrate 60 mg, and calcium. On exam, blood pressure is 100/60 mm Hg, pulse 55. ECG indicates bradycardia with normal intervals. What drug most likely caused her syncope?

A. Citalopram

B. Pravastatin

C. Donepezil

D. Isosorbide

E. Calcium

This woman’s syncope is likely caused by donepezil. Citalopram can lengthen the QT interval, especially in elderly patients, but the normal intervals on ECG eliminate this possibility. Donepezil can cause bradycardia, which can contribute to syncope.

Hernandez and colleagues evaluated a cohort of veterans with dementia over an 8-year period.1 They found that there was a 1.4-fold increased risk of bradycardia in patients with dementia treated with an acetylcholine inhibitor (compared with that in patients who were not taking these medications) and that there was a dose-dependent increase in risk for patients on donepezil.

Park-Wyllie et al. found in a study of 1.4 million older adults a greater than twofold risk of hospitalization for bradycardia in patients treated with a cholinesterase inhibitor.2 Gill and colleagues performed a population-based cohort study of 19,803 elderly patients with dementia who were prescribed cholinesterase inhibitors, and compared them to age-matched controls.3 They found increased hospital visits for syncope in people receiving cholinesterase inhibitors (hazard ratio, 1.76; 95% confidence interval, 1.57-1.98). Other syncope-related events were also more common in people receiving cholinesterase inhibitors, compared with controls: hospital visits for bradycardia (HR, 1.69; 95% CI, 1.32-2.15), permanent pacemaker insertion (HR, 1.49; 95% CI, 1.12-2.00), and hip fracture (HR, 1.18; (95% CI, 1.04-1.34).

Nausea, vomiting, and weight loss are much more common than the rarer side effects of bradycardia and syncope. The frequency of gastroenterological side effects is up to 25%. Cholinesterase inhibitors have modest effects on cognitive function with a high number needed to treat (NNT) of 10, and an NNT as high as 100 for global function. The number needed to harm (NNH) is 4, when gastrointestinal symptoms are added in.4 Another important, problematic side effect of cholinesterase inhibitors is urinary incontinence. This often leads to patients receiving medications, to combat this side effect, that may worsen cognitive function.

Another commonly used medication that scares me in certain circumstances is trimethoprim-sulfamethoxazole. My main concern is when it is used in patients who are elderly, have chronic kidney disease, or are taking other medications that can cause hyperkalemia (ACEIs, ARBs, potassium-sparing diuretics including spironolactone). Hyperkalemia is a real concern in these patient populations. Trimethoprim reduces renal potassium excretion through the competitive inhibition of sodium channels in the distal nephron, in a manner similar to the potassium-sparing diuretic amiloride. Hospitalizations for hyperkalemia are more common in patients who take ACEIs and ARBs and are prescribed trimethoprim-sulfamethoxazole, compared with other antibiotics.5

Sudden cardiac death is also more common in patients who are taking ACEIs or ARBs and receive trimethoprim-sulfamethoxazole.6 Trimethoprim-sulfamethoxazole also has a powerful interaction with warfarin, both displacing warfarin from albumin and inhibiting its metabolism. It raises the INR (international normalized ratio) in warfarin-treated patients much greater than do other antibiotics.7
 

Pearls

  • Think carefully about the use of cholinesterase inhibitors because of the unfavorable NNH vs. NNT.
  • Use caution prescribing trimethoprim for patients who are elderly, especially if they are on an ACEI, an ARB, or spironolactone, and in patients with chronic kidney disease.

Dr. Paauw is professor of medicine in the division of general internal medicine at the University of Washington, Seattle, and he serves as third-year medical student clerkship director at the University of Washington. Contact Dr. Paauw at dpaauw@uw.edu.

References

1. Hernandez RK et al. J Am Geriatr Soc. 2009;57:1997-2003.

2. Park-Wyllie LY et al. PLoS Med. 2009;6:e1000157.

3. Gill SS et al. Arch Intern Med 2009;169:867-73.

4. Peters KR. J Am Geriatr Soc. 2013 Jul;61(7):1170-4.

5. Antoniou TN et al. Arch Intern Med. 2010;170(12):1045-9.

6. Fralick M et al. BMJ. 2014 Oct 30;349:g6196.

7. Glasheen JJ et al. J Gen Intern Med. 2005 Jul;20(7):653-6.

Publications
Publications
Topics
Article Type
Sections
Teambase XML
<?xml version="1.0" encoding="UTF-8"?>
<!--$RCSfile: InCopy_agile.xsl,v $ $Revision: 1.35 $-->
<!--$RCSfile: drupal.xsl,v $ $Revision: 1.7 $-->
<root generator="drupal.xsl" gversion="1.7"> <header> <fileName>163241</fileName> <TBEID>0C049D65.SIG</TBEID> <TBUniqueIdentifier>MD_0C049D65</TBUniqueIdentifier> <newsOrJournal>News</newsOrJournal> <publisherName>Frontline Medical Communications</publisherName> <storyname/> <articleType>353</articleType> <TBLocation>QC Done-All Pubs</TBLocation> <QCDate>20230509T095040</QCDate> <firstPublished>20230509T100028</firstPublished> <LastPublished>20230509T100028</LastPublished> <pubStatus qcode="stat:"/> <embargoDate/> <killDate/> <CMSDate>20230509T100027</CMSDate> <articleSource/> <facebookInfo/> <meetingNumber/> <byline>Douglas S. Paauw</byline> <bylineText>DOUGLAS S. PAAUW, MD</bylineText> <bylineFull>DOUGLAS S. PAAUW, MD</bylineFull> <bylineTitleText/> <USOrGlobal/> <wireDocType/> <newsDocType/> <journalDocType/> <linkLabel/> <pageRange/> <citation/> <quizID/> <indexIssueDate/> <itemClass qcode="ninat:text"/> <provider qcode="provider:imng"> <name>IMNG Medical Media</name> <rightsInfo> <copyrightHolder> <name>Frontline Medical News</name> </copyrightHolder> <copyrightNotice>Copyright (c) 2015 Frontline Medical News, a Frontline Medical Communications Inc. company. All rights reserved. This material may not be published, broadcast, copied, or otherwise reproduced or distributed without the prior written permission of Frontline Medical Communications Inc.</copyrightNotice> </rightsInfo> </provider> <abstract/> <metaDescription>An 85-year-old woman is brought to the emergency department after a syncopal episode. Her caregivers report a similar episode 2 weeks ago, but she recovered so </metaDescription> <articlePDF/> <teaserImage>248215</teaserImage> <teaser>Many medications can cause cardiac symptoms and other serious side effects in elderly patients.</teaser> <title>Medications that scare me</title> <deck/> <disclaimer/> <AuthorList/> <articleURL/> <doi/> <pubMedID/> <publishXMLStatus/> <publishXMLVersion>1</publishXMLVersion> <useEISSN>0</useEISSN> <urgency/> <pubPubdateYear/> <pubPubdateMonth/> <pubPubdateDay/> <pubVolume/> <pubNumber/> <wireChannels/> <primaryCMSID/> <CMSIDs/> <keywords/> <seeAlsos/> <publications_g> <publicationData> <publicationCode>fp</publicationCode> <pubIssueName/> <pubArticleType/> <pubTopics/> <pubCategories/> <pubSections/> </publicationData> <publicationData> <publicationCode>im</publicationCode> <pubIssueName/> <pubArticleType/> <pubTopics/> <pubCategories/> <pubSections/> </publicationData> </publications_g> <publications> <term>15</term> <term canonical="true">21</term> </publications> <sections> <term>52</term> <term canonical="true">39786</term> <term>41022</term> </sections> <topics> <term canonical="true">215</term> </topics> <links> <link> <itemClass qcode="ninat:picture"/> <altRep contenttype="image/jpeg">images/2400c6a0.jpg</altRep> <description role="drol:caption">Dr. Douglas S. Paauw</description> <description role="drol:credit"/> </link> </links> </header> <itemSet> <newsItem> <itemMeta> <itemRole>Main</itemRole> <itemClass>text</itemClass> <title>Medications that scare me</title> <deck/> </itemMeta> <itemContent> <p>An 85-year-old woman is brought to the emergency department after a syncopal episode. Her caregivers report a similar episode 2 weeks ago, but she recovered so quickly they did not seek evaluation for her.</p> <p>[[{"fid":"248215","view_mode":"medstat_image_flush_right","fields":{"format":"medstat_image_flush_right","field_file_image_alt_text[und][0][value]":"Dr. Douglas S. Paauw, University of Washington, Seattle","field_file_image_credit[und][0][value]":"","field_file_image_caption[und][0][value]":"Dr. Douglas S. Paauw"},"type":"media","attributes":{"class":"media-element file-medstat_image_flush_right"}}]]Medications: Omeprazole 20 mg, pravastatin 40 mg, citalopram 10 mg, albuterol, donepezil 10 mg, isosorbide mononitrate 60 mg, and calcium. On exam, blood pressure is 100/60 mm Hg, pulse 55. ECG indicates bradycardia with normal intervals. What drug most likely caused her syncope?<br/><br/><strong>A</strong>. Citalopram<br/><br/><strong>B</strong>. Pravastatin<br/><br/><strong>C</strong>. Donepezil<br/><br/><strong>D</strong>. Isosorbide<br/><br/><strong>E</strong>. Calcium<br/><br/>This woman’s syncope is likely caused by donepezil. Citalopram can lengthen the QT interval, especially in elderly patients, but the normal intervals on ECG eliminate this possibility. Donepezil can cause bradycardia, which can contribute to syncope. <br/><br/>Hernandez and colleagues evaluated a cohort of veterans with dementia over an 8-year period.<sup>1</sup> They found that there was a 1.4-fold increased risk of bradycardia in patients with dementia treated with an acetylcholine inhibitor (compared with that in patients who were not taking these medications) and that there was a dose-dependent increase in risk for patients on donepezil. <br/><br/>Park-Wyllie et al. found in a study of 1.4 million older adults a greater than twofold risk of hospitalization for bradycardia in patients treated with a cholinesterase inhibitor.<sup>2</sup> Gill and colleagues performed a population-based cohort study of 19,803 elderly patients with dementia who were prescribed cholinesterase inhibitors, and compared them to age-matched controls.<sup>3</sup> They found increased hospital visits for syncope in people receiving cholinesterase inhibitors (hazard ratio, 1.76; 95% confidence interval, 1.57-1.98). Other syncope-related events were also more common in people receiving cholinesterase inhibitors, compared with controls: hospital visits for bradycardia (HR, 1.69; 95% CI, 1.32-2.15), permanent pacemaker insertion (HR, 1.49; 95% CI, 1.12-2.00), and hip fracture (HR, 1.18; (95% CI, 1.04-1.34). <br/><br/>Nausea, vomiting, and weight loss are much more common than the rarer side effects of bradycardia and syncope. The frequency of gastroenterological side effects is up to 25%. Cholinesterase inhibitors have modest effects on cognitive function with a high number needed to treat (NNT) of 10, and an NNT as high as 100 for global function. The number needed to harm (NNH) is 4, when gastrointestinal symptoms are added in.<sup>4</sup> Another important, problematic side effect of cholinesterase inhibitors is urinary incontinence. This often leads to patients receiving medications, to combat this side effect, that may worsen cognitive function.<br/><br/>Another commonly used medication that scares me in certain circumstances is trimethoprim-sulfamethoxazole. My main concern is when it is used in patients who are elderly, have chronic kidney disease, or are taking other medications that can cause hyperkalemia (ACEIs, ARBs, potassium-sparing diuretics including spironolactone). Hyperkalemia is a real concern in these patient populations. Trimethoprim reduces renal potassium excretion through the competitive inhibition of sodium channels in the distal nephron, in a manner similar to the potassium-sparing diuretic amiloride. Hospitalizations for hyperkalemia are more common in patients who take ACEIs and ARBs and are prescribed trimethoprim-sulfamethoxazole, compared with other antibiotics.<sup>5</sup> <br/><br/>Sudden cardiac death is also more common in patients who are taking ACEIs or ARBs and receive trimethoprim-sulfamethoxazole.<sup>6 </sup>Trimethoprim-sulfamethoxazole also has a powerful interaction with warfarin, both displacing warfarin from albumin and inhibiting its metabolism. It raises the INR (international normalized ratio) in warfarin-treated patients much greater than do other antibiotics.<sup>7</sup> <br/><br/></p> <h2>Pearls</h2> <ul class="body"> <li>Think carefully about the use of cholinesterase inhibitors because of the unfavorable NNH vs. NNT.</li> <li>Use caution prescribing trimethoprim for patients who are elderly, especially if they are on an ACEI, an ARB, or spironolactone, and in patients with chronic kidney disease.</li> </ul> <p> <em>Dr. Paauw is professor of medicine in the division of general internal medicine at the University of Washington, Seattle, and he serves as third-year medical student clerkship director at the University of Washington. Contact Dr. Paauw at dpaauw@uw.edu.</em> </p> <h2>References</h2> <p>1. Hernandez RK et al. <span class="Hyperlink"><a href="https://agsjournals.onlinelibrary.wiley.com/doi/10.1111/j.1532-5415.2009.02488.x">J Am Geriatr Soc. 2009;57:1997-2003</a></span>.<br/><br/>2. Park-Wyllie LY et al. <span class="Hyperlink"><a href="https://journals.plos.org/plosmedicine/article?id=10.1371/journal.pmed.1000157">PLoS Med. 2009;6:e1000157</a></span>.<br/><br/>3. Gill SS et al. <span class="Hyperlink"><a href="https://jamanetwork.com/journals/jamainternalmedicine/fullarticle/414936">Arch Intern Med 2009;169:867-73</a></span>.<br/><br/>4. Peters KR. <span class="Hyperlink"><a href="https://agsjournals.onlinelibrary.wiley.com/doi/10.1111/jgs.12308">J Am Geriatr Soc. 2013 Jul;61(7):1170-4</a></span>.<br/><br/>5. Antoniou TN et al. <span class="Hyperlink"><a href="https://jamanetwork.com/journals/jamainternalmedicine/fullarticle/416098">Arch Intern Med. 2010;170(12):1045-9.</a></span><br/><br/>6. Fralick M et al. <span class="Hyperlink"><a href="https://www.bmj.com/content/349/bmj.g6196">BMJ. 2014 Oct 30;349:g6196</a></span>.<br/><br/>7. Glasheen JJ et al. <span class="Hyperlink"><a href="https://www.ncbi.nlm.nih.gov/pmc/articles/PMC1490169">J Gen Intern Med. 2005 Jul;20(7):653-6</a></span>.</p> </itemContent> </newsItem> <newsItem> <itemMeta> <itemRole>teaser</itemRole> <itemClass>text</itemClass> <title/> <deck/> </itemMeta> <itemContent> </itemContent> </newsItem> </itemSet></root>
Disallow All Ads
Content Gating
No Gating (article Unlocked/Free)
Alternative CME
Disqus Comments
Default
Use ProPublica
Hide sidebar & use full width
render the right sidebar.
Conference Recap Checkbox
Not Conference Recap
Clinical Edge
Display the Slideshow in this Article
Medscape Article
Display survey writer
Reuters content
Disable Inline Native ads
WebMD Article

What do high BUN/Cr ratios mean?

Article Type
Changed
Thu, 03/09/2023 - 11:07

A 43-year-old man presents to the emergency department with dizziness and a 6-hour history of passing maroon stool. He has been in good health with the only medical problem in his history being depression.

He is taking sertraline. On exam, his blood pressure is 100/60, and his pulse is 100, both while lying down. His blood pressure while standing is 90/60 and his pulse while standing is 130. The rest of his exam is normal. His lab values include hemoglobin of 10, hematocrit of 30, white blood cell of 4.6, platelet count of 175,000, sodium of 142, chloride of 100, bicarbonate of 24, potassium of 3.8, blood urea nitrogen (BUN) of 38, and creatinine clearance (Cr) of 1.1.

Paauw_Doug2_web.jpg
Dr. Paauw

What is the most likely source of his bleeding?

A. Gastric ulcer

B. Meckel’s diverticulum

C. Arteriovenous malformation

D. Diverticulosis

E. Hemorrhoids

What makes the most sense

The most likely cause of this patient’s maroon stool is an upper gastrointestinal bleed, so it would make the most sense for a gastric ulcer to be the source of his bleeding. The clue here is the very high BUN/Cr ratio.

We were all taught early in our training that a high BUN/Cr ratio represented volume depletion. This is certainly the most common cause, but very high BUN/Cr ratios (over 30) can represent causes beyond volume depletion.

Witting and colleagues studied factors that predicted upper GI bleeding in patients presenting without hematemesis. They found that the three strongest predictors were black stool (odds ratio, 16.6), BUN/Cr ratio greater than 30 (OR, 10), and age greater than 50 (OR, 8.4).1

Srygley and colleagues reviewed high-quality studies of factors associated with upper GI bleeding.2 Factors that were found to increase the likelihood of an upper gastrointestinal bleed were Melenic stool on exam (likelihood ratio, 25), blood or coffee grounds on nasogastric aspiration (LR, 9.6), and BUN/Cr ratio greater than 30 (LR, 7.5).

Very high BUN/Cr ratios can indicate problems other than UGI bleeding and volume depletion. High BUN/Cr ratios are seen in patients with heart failure.

Zhang and colleagues studied if a high BUN/Cr ratio helped distinguish heart failure from asthma and chronic obstructive pulmonary disease (COPD).3 They found that, compared with those in the asthma group, the BUN/Cr ratios were significantly increased in the heart failure group (P < .05), whereas no significant differences in BUN/Cr ratios were found between the asthma and COPD groups.

Cheang and colleagues conducted their own study, as well as a meta-analysis, looking to see if high BUN/Cr ratios predicted increased mortality in patients with acute heart failure.4 In the meta-analysis of 8 studies (including their own), they found that the highest BUN/Cr ratio category was associated with an 77% higher all-cause mortality than the lowest category (hazard ratio, 1.77; 95% confidence interval, 1.52-2.07).

High dose corticosteroids can raise BUN levels, especially in patients with chronic kidney disease, and cause unexpectedly high BUN/Cr ratios.
 

Pearl

Very high BUN/Cr ratios (greater than 30) can signify upper GI bleeding, heart failure, or high-dose corticosteroid use.

Dr. Paauw is professor of medicine in the division of general internal medicine at the University of Washington, Seattle, and he serves as third-year medical student clerkship director at the University of Washington. Contact Dr. Paauw at dpaauw@uw.edu.

References

1. Am J Emerg Med. 2006 May;24(3):280-5.

2. JAMA. 2012;307(10):1072-9.

3. Comput Math Methods Med. 2022 Jul 21. doi: 10.1155/2022/4586458.

4. Cardiorenal Med. 2020;10:415-28.

Publications
Topics
Sections

A 43-year-old man presents to the emergency department with dizziness and a 6-hour history of passing maroon stool. He has been in good health with the only medical problem in his history being depression.

He is taking sertraline. On exam, his blood pressure is 100/60, and his pulse is 100, both while lying down. His blood pressure while standing is 90/60 and his pulse while standing is 130. The rest of his exam is normal. His lab values include hemoglobin of 10, hematocrit of 30, white blood cell of 4.6, platelet count of 175,000, sodium of 142, chloride of 100, bicarbonate of 24, potassium of 3.8, blood urea nitrogen (BUN) of 38, and creatinine clearance (Cr) of 1.1.

Paauw_Doug2_web.jpg
Dr. Paauw

What is the most likely source of his bleeding?

A. Gastric ulcer

B. Meckel’s diverticulum

C. Arteriovenous malformation

D. Diverticulosis

E. Hemorrhoids

What makes the most sense

The most likely cause of this patient’s maroon stool is an upper gastrointestinal bleed, so it would make the most sense for a gastric ulcer to be the source of his bleeding. The clue here is the very high BUN/Cr ratio.

We were all taught early in our training that a high BUN/Cr ratio represented volume depletion. This is certainly the most common cause, but very high BUN/Cr ratios (over 30) can represent causes beyond volume depletion.

Witting and colleagues studied factors that predicted upper GI bleeding in patients presenting without hematemesis. They found that the three strongest predictors were black stool (odds ratio, 16.6), BUN/Cr ratio greater than 30 (OR, 10), and age greater than 50 (OR, 8.4).1

Srygley and colleagues reviewed high-quality studies of factors associated with upper GI bleeding.2 Factors that were found to increase the likelihood of an upper gastrointestinal bleed were Melenic stool on exam (likelihood ratio, 25), blood or coffee grounds on nasogastric aspiration (LR, 9.6), and BUN/Cr ratio greater than 30 (LR, 7.5).

Very high BUN/Cr ratios can indicate problems other than UGI bleeding and volume depletion. High BUN/Cr ratios are seen in patients with heart failure.

Zhang and colleagues studied if a high BUN/Cr ratio helped distinguish heart failure from asthma and chronic obstructive pulmonary disease (COPD).3 They found that, compared with those in the asthma group, the BUN/Cr ratios were significantly increased in the heart failure group (P < .05), whereas no significant differences in BUN/Cr ratios were found between the asthma and COPD groups.

Cheang and colleagues conducted their own study, as well as a meta-analysis, looking to see if high BUN/Cr ratios predicted increased mortality in patients with acute heart failure.4 In the meta-analysis of 8 studies (including their own), they found that the highest BUN/Cr ratio category was associated with an 77% higher all-cause mortality than the lowest category (hazard ratio, 1.77; 95% confidence interval, 1.52-2.07).

High dose corticosteroids can raise BUN levels, especially in patients with chronic kidney disease, and cause unexpectedly high BUN/Cr ratios.
 

Pearl

Very high BUN/Cr ratios (greater than 30) can signify upper GI bleeding, heart failure, or high-dose corticosteroid use.

Dr. Paauw is professor of medicine in the division of general internal medicine at the University of Washington, Seattle, and he serves as third-year medical student clerkship director at the University of Washington. Contact Dr. Paauw at dpaauw@uw.edu.

References

1. Am J Emerg Med. 2006 May;24(3):280-5.

2. JAMA. 2012;307(10):1072-9.

3. Comput Math Methods Med. 2022 Jul 21. doi: 10.1155/2022/4586458.

4. Cardiorenal Med. 2020;10:415-28.

A 43-year-old man presents to the emergency department with dizziness and a 6-hour history of passing maroon stool. He has been in good health with the only medical problem in his history being depression.

He is taking sertraline. On exam, his blood pressure is 100/60, and his pulse is 100, both while lying down. His blood pressure while standing is 90/60 and his pulse while standing is 130. The rest of his exam is normal. His lab values include hemoglobin of 10, hematocrit of 30, white blood cell of 4.6, platelet count of 175,000, sodium of 142, chloride of 100, bicarbonate of 24, potassium of 3.8, blood urea nitrogen (BUN) of 38, and creatinine clearance (Cr) of 1.1.

Paauw_Doug2_web.jpg
Dr. Paauw

What is the most likely source of his bleeding?

A. Gastric ulcer

B. Meckel’s diverticulum

C. Arteriovenous malformation

D. Diverticulosis

E. Hemorrhoids

What makes the most sense

The most likely cause of this patient’s maroon stool is an upper gastrointestinal bleed, so it would make the most sense for a gastric ulcer to be the source of his bleeding. The clue here is the very high BUN/Cr ratio.

We were all taught early in our training that a high BUN/Cr ratio represented volume depletion. This is certainly the most common cause, but very high BUN/Cr ratios (over 30) can represent causes beyond volume depletion.

Witting and colleagues studied factors that predicted upper GI bleeding in patients presenting without hematemesis. They found that the three strongest predictors were black stool (odds ratio, 16.6), BUN/Cr ratio greater than 30 (OR, 10), and age greater than 50 (OR, 8.4).1

Srygley and colleagues reviewed high-quality studies of factors associated with upper GI bleeding.2 Factors that were found to increase the likelihood of an upper gastrointestinal bleed were Melenic stool on exam (likelihood ratio, 25), blood or coffee grounds on nasogastric aspiration (LR, 9.6), and BUN/Cr ratio greater than 30 (LR, 7.5).

Very high BUN/Cr ratios can indicate problems other than UGI bleeding and volume depletion. High BUN/Cr ratios are seen in patients with heart failure.

Zhang and colleagues studied if a high BUN/Cr ratio helped distinguish heart failure from asthma and chronic obstructive pulmonary disease (COPD).3 They found that, compared with those in the asthma group, the BUN/Cr ratios were significantly increased in the heart failure group (P < .05), whereas no significant differences in BUN/Cr ratios were found between the asthma and COPD groups.

Cheang and colleagues conducted their own study, as well as a meta-analysis, looking to see if high BUN/Cr ratios predicted increased mortality in patients with acute heart failure.4 In the meta-analysis of 8 studies (including their own), they found that the highest BUN/Cr ratio category was associated with an 77% higher all-cause mortality than the lowest category (hazard ratio, 1.77; 95% confidence interval, 1.52-2.07).

High dose corticosteroids can raise BUN levels, especially in patients with chronic kidney disease, and cause unexpectedly high BUN/Cr ratios.
 

Pearl

Very high BUN/Cr ratios (greater than 30) can signify upper GI bleeding, heart failure, or high-dose corticosteroid use.

Dr. Paauw is professor of medicine in the division of general internal medicine at the University of Washington, Seattle, and he serves as third-year medical student clerkship director at the University of Washington. Contact Dr. Paauw at dpaauw@uw.edu.

References

1. Am J Emerg Med. 2006 May;24(3):280-5.

2. JAMA. 2012;307(10):1072-9.

3. Comput Math Methods Med. 2022 Jul 21. doi: 10.1155/2022/4586458.

4. Cardiorenal Med. 2020;10:415-28.

Publications
Publications
Topics
Article Type
Sections
Teambase XML
<?xml version="1.0" encoding="UTF-8"?>
<!--$RCSfile: InCopy_agile.xsl,v $ $Revision: 1.35 $-->
<!--$RCSfile: drupal.xsl,v $ $Revision: 1.7 $-->
<root generator="drupal.xsl" gversion="1.7"> <header> <fileName>162570</fileName> <TBEID>0C048DDD.SIG</TBEID> <TBUniqueIdentifier>MD_0C048DDD</TBUniqueIdentifier> <newsOrJournal>News</newsOrJournal> <publisherName>Frontline Medical Communications</publisherName> <storyname/> <articleType>2</articleType> <TBLocation>QC Done-All Pubs</TBLocation> <QCDate>20230309T104103</QCDate> <firstPublished>20230309T110048</firstPublished> <LastPublished>20230309T110048</LastPublished> <pubStatus qcode="stat:"/> <embargoDate/> <killDate/> <CMSDate>20230309T110048</CMSDate> <articleSource/> <facebookInfo/> <meetingNumber/> <byline/> <bylineText>DOUGLAS S. PAAUW, MD</bylineText> <bylineFull>DOUGLAS S. PAAUW, MD</bylineFull> <bylineTitleText>MDedge News</bylineTitleText> <USOrGlobal/> <wireDocType/> <newsDocType>Column</newsDocType> <journalDocType/> <linkLabel/> <pageRange/> <citation/> <quizID/> <indexIssueDate/> <itemClass qcode="ninat:text"/> <provider qcode="provider:imng"> <name>IMNG Medical Media</name> <rightsInfo> <copyrightHolder> <name>Frontline Medical News</name> </copyrightHolder> <copyrightNotice>Copyright (c) 2015 Frontline Medical News, a Frontline Medical Communications Inc. company. All rights reserved. This material may not be published, broadcast, copied, or otherwise reproduced or distributed without the prior written permission of Frontline Medical Communications Inc.</copyrightNotice> </rightsInfo> </provider> <abstract/> <metaDescription>A 43-year-old man presents to the emergency department with dizziness and a 6-hour history of passing maroon stool.</metaDescription> <articlePDF/> <teaserImage>241147</teaserImage> <teaser>What is the most likely source of this patient’s bleeding? </teaser> <title>What do high BUN/Cr ratios mean?</title> <deck/> <disclaimer/> <AuthorList/> <articleURL/> <doi/> <pubMedID/> <publishXMLStatus/> <publishXMLVersion>1</publishXMLVersion> <useEISSN>0</useEISSN> <urgency/> <pubPubdateYear/> <pubPubdateMonth/> <pubPubdateDay/> <pubVolume/> <pubNumber/> <wireChannels/> <primaryCMSID/> <CMSIDs/> <keywords/> <seeAlsos/> <publications_g> <publicationData> <publicationCode>im</publicationCode> <pubIssueName/> <pubArticleType/> <pubTopics/> <pubCategories/> <pubSections/> </publicationData> <publicationData> <publicationCode>fp</publicationCode> <pubIssueName/> <pubArticleType/> <pubTopics/> <pubCategories/> <pubSections/> </publicationData> <publicationData> <publicationCode>endo</publicationCode> <pubIssueName/> <pubArticleType/> <pubTopics/> <pubCategories/> <pubSections/> </publicationData> <publicationData> <publicationCode>GIHOLD</publicationCode> <pubIssueName>January 2014</pubIssueName> <pubArticleType/> <pubTopics/> <pubCategories/> <pubSections/> <journalTitle/> <journalFullTitle/> <copyrightStatement/> </publicationData> </publications_g> <publications> <term canonical="true">21</term> <term>15</term> <term>34</term> </publications> <sections> <term canonical="true">39786</term> <term>52</term> <term>41022</term> </sections> <topics> <term canonical="true">213</term> <term>255</term> <term>277</term> </topics> <links> <link> <itemClass qcode="ninat:picture"/> <altRep contenttype="image/jpeg">images/2400b8e1.jpg</altRep> <description role="drol:caption">Dr. Paauw</description> <description role="drol:credit"/> </link> </links> </header> <itemSet> <newsItem> <itemMeta> <itemRole>Main</itemRole> <itemClass>text</itemClass> <title>What do high BUN/Cr ratios mean?</title> <deck/> </itemMeta> <itemContent> <p><span class="tag metaDescription">A 43-year-old man presents to the emergency department with dizziness and a 6-hour history of passing maroon stool.</span> He has been in good health with the only medical problem in his history being depression. </p> <p>He is taking sertraline. On exam, his blood pressure is 100/60, and his pulse is 100, both while lying down. His blood pressure while standing is 90/60 and his pulse while standing is 130. The rest of his exam is normal. His lab values include hemoglobin of 10, hematocrit of 30, white blood cell of 4.6, platelet count of 175,000, sodium of 142, chloride of 100, bicarbonate of 24, potassium of 3.8, blood urea nitrogen (BUN) of 38, and creatinine clearance (Cr) of 1.1.<br/><br/>[[{"fid":"241147","view_mode":"medstat_image_flush_left","fields":{"format":"medstat_image_flush_left","field_file_image_alt_text[und][0][value]":"Dr. Paauw","field_file_image_credit[und][0][value]":"","field_file_image_caption[und][0][value]":"Dr. Paauw"},"type":"media","attributes":{"class":"media-element file-medstat_image_flush_left"}}]]What is the most likely source of his bleeding?</p> <p><strong>A.</strong> Gastric ulcer<br/><br/><strong>B.</strong> Meckel’s diverticulum<br/><br/><strong>C.</strong> Arteriovenous malformation<br/><br/><strong>D. </strong>Diverticulosis<br/><br/><strong>E. </strong>Hemorrhoids</p> <h2>What makes the most sense</h2> <p>The most likely cause of this patient’s maroon stool is an upper gastrointestinal bleed, so it would make the most sense for a gastric ulcer to be the source of his bleeding. The clue here is the very high BUN/Cr ratio. </p> <p>We were all taught early in our training that a high BUN/Cr ratio represented volume depletion. This is certainly the most common cause, but very high BUN/Cr ratios (over 30) can represent causes beyond volume depletion. <br/><br/>Witting and colleagues studied factors that predicted upper GI bleeding in patients presenting without hematemesis. They found that the three strongest predictors were black stool (odds ratio, 16.6), BUN/Cr ratio greater than 30 (OR, 10), and age greater than 50 (OR, 8.4).<sup>1</sup> <br/><br/>Srygley and colleagues reviewed high-quality studies of factors associated with upper GI bleeding.<sup>2</sup> Factors that were found to increase the likelihood of an upper gastrointestinal bleed were Melenic stool on exam (likelihood ratio, 25), blood or coffee grounds on nasogastric aspiration (LR, 9.6), and BUN/Cr ratio greater than 30 (LR, 7.5). <br/><br/>Very high BUN/Cr ratios can indicate problems other than UGI bleeding and volume depletion. High BUN/Cr ratios are seen in patients with heart failure. <br/><br/>Zhang and colleagues studied if a high BUN/Cr ratio helped distinguish heart failure from asthma and chronic obstructive pulmonary disease (COPD).<sup>3</sup> They found that, compared with those in the asthma group, the BUN/Cr ratios were significantly increased in the heart failure group (<em>P</em> &lt; .05), whereas no significant differences in BUN/Cr ratios were found between the asthma and COPD groups. <br/><br/>Cheang and colleagues conducted their own study, as well as a meta-analysis, looking to see if high BUN/Cr ratios predicted increased mortality in patients with acute heart failure.<sup>4</sup> In the meta-analysis of 8 studies (including their own), they found that the highest BUN/Cr ratio category was associated with an 77% higher all-cause mortality than the lowest category (hazard ratio, 1.77; 95% confidence interval, 1.52-2.07). <br/><br/>High dose corticosteroids can raise BUN levels, especially in patients with chronic kidney disease, and cause unexpectedly high BUN/Cr ratios.<br/><br/></p> <h2>Pearl</h2> <p>Very high BUN/Cr ratios (greater than 30) can signify upper GI bleeding, heart failure, or high-dose corticosteroid use.</p> <p> <em>Dr. Paauw is professor of medicine in the division of general internal medicine at the University of Washington, Seattle, and he serves as third-year medical student clerkship director at the University of Washington. Contact Dr. Paauw at dpaauw@uw.edu.</em> </p> <h2>References</h2> <p>1. Am J Emerg Med. 2006 May;24(3):280-5.<br/><br/>2. JAMA. 2012;307(10):1072-9.<br/><br/>3. Comput Math Methods Med. 2022 Jul 21. doi: 10.1155/2022/4586458.<br/><br/>4. Cardiorenal Med. 2020;10:415-28.</p> </itemContent> </newsItem> <newsItem> <itemMeta> <itemRole>teaser</itemRole> <itemClass>text</itemClass> <title/> <deck/> </itemMeta> <itemContent> </itemContent> </newsItem> </itemSet></root>
Disallow All Ads
Content Gating
No Gating (article Unlocked/Free)
Alternative CME
Disqus Comments
Default
Use ProPublica
Hide sidebar & use full width
render the right sidebar.
Conference Recap Checkbox
Not Conference Recap
Clinical Edge
Display the Slideshow in this Article
Medscape Article
Display survey writer
Reuters content
Disable Inline Native ads
WebMD Article

How to talk with patients in ways that help them feel heard and understood

Article Type
Changed
Thu, 01/19/2023 - 16:24

A common sentiment shared by patients who are happy with their health care professionals is, “I feel heard and understood.” How do we become those professionals and make sure that we are doing a good job connecting and communicating with our patients?

Paauw_Doug_SEATTLE_2019_web.jpg
Dr. Douglas S. Paauw

Here are a few suggestions on how to do this.
 

Practice intent listening

When a patient shares their symptoms with you, show genuine curiosity and concern. Ask clarifying questions. Ask how the symptom or problem is affecting their day-to-day life. Avoid quick, rapid-fire questions back at the patient. Do not accept a patient self-diagnosis.

When a patient with a first-time headache says they are having a migraine headache, for example, ask many clarifying questions to make sure you can make a diagnosis of headache type, then use all the information you have gathered to educate the patient on what you believe they have.

It is easy to jump to treatment, but we always want to make sure we have the diagnosis correct first. By intently listening, it also makes it much easier to tell a patient you do not know what is causing their symptoms, but that you and the patient will be vigilant for any future clues that may lead to a diagnosis.
 

Use terminology that patients understand

Rachael Gotlieb, MD, and colleagues published an excellent study with eye-opening results on common phrases we use as health care providers and how often patients do not understand them.

Only 9% of patients understood what was meant when they were asked if they have been febrile. Only 2% understood what was meant by “I am concerned the patient has an occult infection.” Only 21% understood that “your xray findings were quite impressive” was bad news.

It is easy to avoid these medical language traps, we just have to check our doctor speak. Ask, “Do you have a fever?” Say, “I am concerned you may have an infection that is hard to find.”

Several other terms we use all the time in explaining things to patients that I have found most patients do not understand are the terms bilateral, systemic, and significant. Think carefully as you explain things to patients and check back to have them repeat to you what they think you said.
 

Be comfortable saying you don’t know

Many symptoms in medicine end up not being diagnosable. When a patient shares symptoms that do not fit a pattern of a disease, it is important to share with them why you think it is okay to wait and watch, even if you do not have a diagnosis.

Patients find it comforting that you are so honest with them. Doing this also has the benefit of gaining patients’ trust when you are sure about something, because it tells them you don’t have an answer for everything.
 

Ask your patients what they think is causing their symptoms

This way, you know what their big fear is. You can address what they are worried about, even if it isn’t something you are considering.

Patients are often fearful of a disease a close friend or relative has, so when they get new symptoms, they fear diseases that we might not think of. By knowing what they are fearful of, you can reassure when appropriate.

Dr. Paauw is professor of medicine in the division of general internal medicine at the University of Washington, Seattle, and he serves as third-year medical student clerkship director at the University of Washington. Contact Dr. Paauw at dpaauw@uw.edu.

Publications
Topics
Sections

A common sentiment shared by patients who are happy with their health care professionals is, “I feel heard and understood.” How do we become those professionals and make sure that we are doing a good job connecting and communicating with our patients?

Paauw_Doug_SEATTLE_2019_web.jpg
Dr. Douglas S. Paauw

Here are a few suggestions on how to do this.
 

Practice intent listening

When a patient shares their symptoms with you, show genuine curiosity and concern. Ask clarifying questions. Ask how the symptom or problem is affecting their day-to-day life. Avoid quick, rapid-fire questions back at the patient. Do not accept a patient self-diagnosis.

When a patient with a first-time headache says they are having a migraine headache, for example, ask many clarifying questions to make sure you can make a diagnosis of headache type, then use all the information you have gathered to educate the patient on what you believe they have.

It is easy to jump to treatment, but we always want to make sure we have the diagnosis correct first. By intently listening, it also makes it much easier to tell a patient you do not know what is causing their symptoms, but that you and the patient will be vigilant for any future clues that may lead to a diagnosis.
 

Use terminology that patients understand

Rachael Gotlieb, MD, and colleagues published an excellent study with eye-opening results on common phrases we use as health care providers and how often patients do not understand them.

Only 9% of patients understood what was meant when they were asked if they have been febrile. Only 2% understood what was meant by “I am concerned the patient has an occult infection.” Only 21% understood that “your xray findings were quite impressive” was bad news.

It is easy to avoid these medical language traps, we just have to check our doctor speak. Ask, “Do you have a fever?” Say, “I am concerned you may have an infection that is hard to find.”

Several other terms we use all the time in explaining things to patients that I have found most patients do not understand are the terms bilateral, systemic, and significant. Think carefully as you explain things to patients and check back to have them repeat to you what they think you said.
 

Be comfortable saying you don’t know

Many symptoms in medicine end up not being diagnosable. When a patient shares symptoms that do not fit a pattern of a disease, it is important to share with them why you think it is okay to wait and watch, even if you do not have a diagnosis.

Patients find it comforting that you are so honest with them. Doing this also has the benefit of gaining patients’ trust when you are sure about something, because it tells them you don’t have an answer for everything.
 

Ask your patients what they think is causing their symptoms

This way, you know what their big fear is. You can address what they are worried about, even if it isn’t something you are considering.

Patients are often fearful of a disease a close friend or relative has, so when they get new symptoms, they fear diseases that we might not think of. By knowing what they are fearful of, you can reassure when appropriate.

Dr. Paauw is professor of medicine in the division of general internal medicine at the University of Washington, Seattle, and he serves as third-year medical student clerkship director at the University of Washington. Contact Dr. Paauw at dpaauw@uw.edu.

A common sentiment shared by patients who are happy with their health care professionals is, “I feel heard and understood.” How do we become those professionals and make sure that we are doing a good job connecting and communicating with our patients?

Paauw_Doug_SEATTLE_2019_web.jpg
Dr. Douglas S. Paauw

Here are a few suggestions on how to do this.
 

Practice intent listening

When a patient shares their symptoms with you, show genuine curiosity and concern. Ask clarifying questions. Ask how the symptom or problem is affecting their day-to-day life. Avoid quick, rapid-fire questions back at the patient. Do not accept a patient self-diagnosis.

When a patient with a first-time headache says they are having a migraine headache, for example, ask many clarifying questions to make sure you can make a diagnosis of headache type, then use all the information you have gathered to educate the patient on what you believe they have.

It is easy to jump to treatment, but we always want to make sure we have the diagnosis correct first. By intently listening, it also makes it much easier to tell a patient you do not know what is causing their symptoms, but that you and the patient will be vigilant for any future clues that may lead to a diagnosis.
 

Use terminology that patients understand

Rachael Gotlieb, MD, and colleagues published an excellent study with eye-opening results on common phrases we use as health care providers and how often patients do not understand them.

Only 9% of patients understood what was meant when they were asked if they have been febrile. Only 2% understood what was meant by “I am concerned the patient has an occult infection.” Only 21% understood that “your xray findings were quite impressive” was bad news.

It is easy to avoid these medical language traps, we just have to check our doctor speak. Ask, “Do you have a fever?” Say, “I am concerned you may have an infection that is hard to find.”

Several other terms we use all the time in explaining things to patients that I have found most patients do not understand are the terms bilateral, systemic, and significant. Think carefully as you explain things to patients and check back to have them repeat to you what they think you said.
 

Be comfortable saying you don’t know

Many symptoms in medicine end up not being diagnosable. When a patient shares symptoms that do not fit a pattern of a disease, it is important to share with them why you think it is okay to wait and watch, even if you do not have a diagnosis.

Patients find it comforting that you are so honest with them. Doing this also has the benefit of gaining patients’ trust when you are sure about something, because it tells them you don’t have an answer for everything.
 

Ask your patients what they think is causing their symptoms

This way, you know what their big fear is. You can address what they are worried about, even if it isn’t something you are considering.

Patients are often fearful of a disease a close friend or relative has, so when they get new symptoms, they fear diseases that we might not think of. By knowing what they are fearful of, you can reassure when appropriate.

Dr. Paauw is professor of medicine in the division of general internal medicine at the University of Washington, Seattle, and he serves as third-year medical student clerkship director at the University of Washington. Contact Dr. Paauw at dpaauw@uw.edu.

Publications
Publications
Topics
Article Type
Sections
Teambase XML
<?xml version="1.0" encoding="UTF-8"?>
<!--$RCSfile: InCopy_agile.xsl,v $ $Revision: 1.35 $-->
<!--$RCSfile: drupal.xsl,v $ $Revision: 1.7 $-->
<root generator="drupal.xsl" gversion="1.7"> <header> <fileName>161839</fileName> <TBEID>0C047C89.SIG</TBEID> <TBUniqueIdentifier>MD_0C047C89</TBUniqueIdentifier> <newsOrJournal>News</newsOrJournal> <publisherName>Frontline Medical Communications</publisherName> <storyname/> <articleType>353</articleType> <TBLocation>QC Done-All Pubs</TBLocation> <QCDate>20230118T151537</QCDate> <firstPublished>20230118T151901</firstPublished> <LastPublished>20230118T151901</LastPublished> <pubStatus qcode="stat:"/> <embargoDate/> <killDate/> <CMSDate>20230118T151901</CMSDate> <articleSource/> <facebookInfo/> <meetingNumber/> <byline/> <bylineText>DOUGLAS S. PAAUW, MD</bylineText> <bylineFull>DOUGLAS S. PAAUW, MD</bylineFull> <bylineTitleText/> <USOrGlobal/> <wireDocType/> <newsDocType>Opinion</newsDocType> <journalDocType/> <linkLabel/> <pageRange/> <citation/> <quizID/> <indexIssueDate/> <itemClass qcode="ninat:text"/> <provider qcode="provider:imng"> <name>IMNG Medical Media</name> <rightsInfo> <copyrightHolder> <name>Frontline Medical News</name> </copyrightHolder> <copyrightNotice>Copyright (c) 2015 Frontline Medical News, a Frontline Medical Communications Inc. company. All rights reserved. This material may not be published, broadcast, copied, or otherwise reproduced or distributed without the prior written permission of Frontline Medical Communications Inc.</copyrightNotice> </rightsInfo> </provider> <abstract/> <metaDescription>A common sentiment shared by patients who are happy with their health care professionals is, “I feel heard and understood.”</metaDescription> <articlePDF/> <teaserImage>248215</teaserImage> <teaser>How do we make sure that we are doing a good job connecting and communicating with our patients?</teaser> <title>How to talk with patients in ways that help them feel heard and understood</title> <deck/> <disclaimer/> <AuthorList/> <articleURL/> <doi/> <pubMedID/> <publishXMLStatus/> <publishXMLVersion>1</publishXMLVersion> <useEISSN>0</useEISSN> <urgency/> <pubPubdateYear/> <pubPubdateMonth/> <pubPubdateDay/> <pubVolume/> <pubNumber/> <wireChannels/> <primaryCMSID/> <CMSIDs/> <keywords/> <seeAlsos/> <publications_g> <publicationData> <publicationCode>im</publicationCode> <pubIssueName/> <pubArticleType/> <pubTopics/> <pubCategories/> <pubSections/> </publicationData> <publicationData> <publicationCode>fp</publicationCode> <pubIssueName/> <pubArticleType/> <pubTopics/> <pubCategories/> <pubSections/> </publicationData> <publicationData> <publicationCode>chph</publicationCode> <pubIssueName/> <pubArticleType/> <pubTopics/> <pubCategories/> <pubSections/> </publicationData> <publicationData> <publicationCode>card</publicationCode> <pubIssueName/> <pubArticleType/> <pubTopics/> <pubCategories/> <pubSections/> </publicationData> <publicationData> <publicationCode>cpn</publicationCode> <pubIssueName/> <pubArticleType/> <pubTopics/> <pubCategories/> <pubSections/> </publicationData> <publicationData> <publicationCode>endo</publicationCode> <pubIssueName/> <pubArticleType/> <pubTopics/> <pubCategories/> <pubSections/> </publicationData> <publicationData> <publicationCode>skin</publicationCode> <pubIssueName/> <pubArticleType/> <pubTopics/> <pubCategories/> <pubSections/> </publicationData> <publicationData> <publicationCode>hemn</publicationCode> <pubIssueName/> <pubArticleType/> <pubTopics/> <pubCategories/> <pubSections/> </publicationData> <publicationData> <publicationCode>idprac</publicationCode> <pubIssueName/> <pubArticleType/> <pubTopics/> <pubCategories/> <pubSections/> </publicationData> <publicationData> <publicationCode>mdemed</publicationCode> <pubIssueName/> <pubArticleType/> <pubTopics/> <pubCategories/> <pubSections/> <journalTitle/> <journalFullTitle/> <copyrightStatement/> </publicationData> <publicationData> <publicationCode>mdsurg</publicationCode> <pubIssueName/> <pubArticleType/> <pubTopics/> <pubCategories/> <pubSections/> <journalTitle/> <journalFullTitle/> <copyrightStatement>2018 Frontline Medical Communications Inc.,</copyrightStatement> </publicationData> <publicationData> <publicationCode>nr</publicationCode> <pubIssueName/> <pubArticleType/> <pubTopics/> <pubCategories/> <pubSections/> <journalTitle>Neurology Reviews</journalTitle> <journalFullTitle>Neurology Reviews</journalFullTitle> <copyrightStatement>2018 Frontline Medical Communications Inc.,</copyrightStatement> </publicationData> <publicationData> <publicationCode>ob</publicationCode> <pubIssueName/> <pubArticleType/> <pubTopics/> <pubCategories/> <pubSections/> </publicationData> <publicationData> <publicationCode>oncr</publicationCode> <pubIssueName/> <pubArticleType/> <pubTopics/> <pubCategories/> <pubSections/> </publicationData> <publicationData> <publicationCode>pn</publicationCode> <pubIssueName/> <pubArticleType/> <pubTopics/> <pubCategories/> <pubSections/> </publicationData> <publicationData> <publicationCode>rn</publicationCode> <pubIssueName/> <pubArticleType/> <pubTopics/> <pubCategories/> <pubSections/> </publicationData> </publications_g> <publications> <term canonical="true">21</term> <term>15</term> <term>6</term> <term>5</term> <term>9</term> <term>34</term> <term>13</term> <term>18</term> <term>20</term> <term>58877</term> <term>52226</term> <term>22</term> <term>23</term> <term>31</term> <term>25</term> <term>26</term> </publications> <sections> <term>52</term> <term>41022</term> <term canonical="true">39786</term> </sections> <topics> <term canonical="true">38029</term> <term>258</term> <term>27442</term> <term>278</term> </topics> <links> <link> <itemClass qcode="ninat:picture"/> <altRep contenttype="image/jpeg">images/2400c6a0.jpg</altRep> <description role="drol:caption">Dr. Douglas S. Paauw</description> <description role="drol:credit"/> </link> </links> </header> <itemSet> <newsItem> <itemMeta> <itemRole>Main</itemRole> <itemClass>text</itemClass> <title>How to talk with patients in ways that help them feel heard and understood</title> <deck/> </itemMeta> <itemContent> <p><span class="tag metaDescription">A common sentiment shared by patients who are happy with their health care professionals is, “I feel heard and understood.”</span> How do we become those professionals and make sure that we are doing a good job connecting and communicating with our patients?[[{"fid":"248215","view_mode":"medstat_image_flush_right","fields":{"format":"medstat_image_flush_right","field_file_image_alt_text[und][0][value]":"Dr. Douglas S. Paauw, University of Washington, Seattle","field_file_image_credit[und][0][value]":"","field_file_image_caption[und][0][value]":"Dr. Douglas S. Paauw"},"type":"media","attributes":{"class":"media-element file-medstat_image_flush_right"}}]]</p> <p>Here are a few suggestions on how to do this.<br/><br/></p> <h2>Practice intent listening</h2> <p>When a patient shares their symptoms with you, show genuine curiosity and concern. Ask clarifying questions. Ask how the symptom or problem is affecting their day-to-day life. Avoid quick, rapid-fire questions back at the patient. Do not accept a patient self-diagnosis. </p> <p>When a patient with a first-time headache says they are having a migraine headache, for example, ask many clarifying questions to make sure you can make a diagnosis of headache type, then use all the information you have gathered to educate the patient on what you believe they have. <br/><br/>It is easy to jump to treatment, but we always want to make sure we have the diagnosis correct first. By intently listening, it also makes it much easier to tell a patient you do not know what is causing their symptoms, but that you and the patient will be vigilant for any future clues that may lead to a diagnosis.<br/><br/></p> <h2>Use terminology that patients understand</h2> <p>Rachael Gotlieb, MD, and colleagues published an excellent <a href="https://jamanetwork.com/journals/jamanetworkopen/fullarticle/2799079">study</a> with eye-opening results on common phrases we use as health care providers and how often patients do not understand them.</p> <p>Only 9% of patients understood what was meant when they were asked if they have been febrile. Only 2% understood what was meant by “I am concerned the patient has an occult infection.” Only 21% understood that “your xray findings were quite impressive” was bad news. <br/><br/>It is easy to avoid these medical language traps, we just have to check our doctor speak. Ask, “Do you have a fever?” Say, “I am concerned you may have an infection that is hard to find.”<br/><br/>Several other terms we use all the time in explaining things to patients that I have found most patients do not understand are the terms bilateral, systemic, and significant. Think carefully as you explain things to patients and check back to have them repeat to you what they think you said.<br/><br/></p> <h2>Be comfortable saying you don’t know</h2> <p>Many symptoms in medicine end up not being diagnosable. When a patient shares symptoms that do not fit a pattern of a disease, it is important to share with them why you think it is okay to wait and watch, even if you do not have a diagnosis. </p> <p>Patients find it comforting that you are so honest with them. Doing this also has the benefit of gaining patients’ trust when you are sure about something, because it tells them you don’t have an answer for everything.<br/><br/></p> <h2>Ask your patients what they think is causing their symptoms </h2> <p>This way, you know what their big fear is. You can address what they are worried about, even if it isn’t something you are considering.</p> <p>Patients are often fearful of a disease a close friend or relative has, so when they get new symptoms, they fear diseases that we might not think of. By knowing what they are fearful of, you can reassure when appropriate.<span class="end"/></p> <p> <em>Dr. Paauw is professor of medicine in the division of general internal medicine at the University of Washington, Seattle, and he serves as third-year medical student clerkship director at the University of Washington. Contact Dr. Paauw at <a href="mailto:dpaauw%40uw.edu?subject=">dpaauw@uw.edu</a>.</em> </p> </itemContent> </newsItem> <newsItem> <itemMeta> <itemRole>teaser</itemRole> <itemClass>text</itemClass> <title/> <deck/> </itemMeta> <itemContent> </itemContent> </newsItem> </itemSet></root>
Disallow All Ads
Content Gating
No Gating (article Unlocked/Free)
Alternative CME
Disqus Comments
Default
Use ProPublica
Hide sidebar & use full width
render the right sidebar.
Conference Recap Checkbox
Not Conference Recap
Clinical Edge
Display the Slideshow in this Article
Medscape Article
Display survey writer
Reuters content
Disable Inline Native ads
WebMD Article

How can I keep from losing my mind?

Article Type
Changed
Wed, 11/09/2022 - 10:43

A 67-year-old man asks about what he can do to prevent dementia. He reports his mother had dementia, and he wants to do everything he can to prevent it. Which of the following has evidence of benefit?

A) Thiamine

B) Vitamin E

C) Multivitamin (MV)

D) Keto diet

E) Red wine
 

FDA-approved therapies for dementia

To date the actual therapies for dementia have been disappointing. Donepezil, the most prescribed medication for the treatment of dementia has a number-needed-to treat (NNT) over 17, and causes frequent side effects. Aducanumab was recently approved by the Food and Drug Administration for the treatment of Alzheimer’s disease (AD), but controversy has arisen, as the clinical results were modest, and the price tag will be large – estimated at $30,000-$50,000/year.

Preventive options that may decrease the likelihood of dementia

Patients often ask the question stated above. Regarding how to respond to that question, choice C, MV, has some recent evidence of benefit. Baker and colleagues studied the effect of cocoa extract and multivitamins on cognitive function in the COSMOS-Mind trial.1 A total of 2,262 people were enrolled, and over 90% completed baseline and at least one annual cognitive assessment. Cocoa extract had no impact on global cognition (confidence interval [CI], –.02-.08, P = .28), but MV supplementation did have a statistically significant impact on global cognition (CI, .02-.12, P less than .007).

Paauw_Doug_SEATTLE_2019_web.jpg
Dr. Douglas S. Paauw

Vitamin E has been enthusiastically endorsed in the past as a treatment to prevent cognitive decline. The most recent Cochrane review on vitamin E concluded there was no evidence that the alpha-tocopherol form of vitamin E given to people with MCI prevents progression to dementia, or that it improves cognitive function in people with MCI or dementia due to AD.2

Exercise has long been a mainstay of our advice to patients as something they can do to help prevent dementia. Yu and colleagues did a meta-analysis of almost 400 randomized controlled trials and observational studies to grade the evidence on different interventions.3 They gave exercise a grade B for evidence of benefit.

A recent study addressed this issue, and I think it is helpful on quantifying how much exercise is needed. Del Pozo Cruz and colleagues did a prospective population-based cohort study of 78,000 adults aged 40-79, with an average of 6.9 years of follow up.4 The optimal step count was 9,826 steps (hazard ratio [HR], 0.49; 95% CI, 0.39-0.62) and the minimal step count for benefit was 3,826 steps (HR, 0.75; 95% CI, 0.67-0.83).
 

Modifiable factors

The other major modifiable factors to consider are problems with special senses. Both vision loss and hearing loss have been associated with cognitive impairment.

Shang and colleagues published a meta-analysis of 14 cohort studies addressing vision impairment and cognitive function involving more than 6 million individuals.5 They concluded that vision impairment is associated with an increased risk of both dementia and cognitive impairment in older adults.

Loughrey and colleagues performed a meta-analysis of 36 studies addressing hearing loss and cognitive decline.6 They reported that, among cross-sectional studies, a significant association was found for cognitive impairment (odds ratio [OR], 2.00; 95% CI, 1.39-2.89) and dementia (OR, 2.42; 95% CI, 1.24-4.72). A similar finding was present in prospective cohort studies with a significant association being found for cognitive impairment (OR, 1.22; 95% CI, 1.09-1.36) and dementia (OR, 1.28; 95% CI, 1.02-1.59).

A 25-year prospective, population-based study of patients with hearing loss revealed a difference in the rate of change in MMSE score over the 25-year follow-up between participants with hearing loss not using hearing aids matched with controls who didn’t have hearing loss. Those with untreated hearing loss had more cognitive decline than that of patients without hearing loss.7 The subjects with hearing loss using a hearing aid had no difference in cognitive decline from controls.
 

 

 

Pearl

Several simple and safe interventions may protect our patients from cognitive decline. These include taking a daily multivitamin, walking more than 4,000 steps a day, and optimizing vision and hearing.

Dr. Paauw is professor of medicine in the division of general internal medicine at the University of Washington, Seattle, and he serves as third-year medical student clerkship director at the University of Washington. Contact Dr. Paauw at dpaauw@uw.edu.

References

1. Baker LD et al. Effects of cocoa extract and a multivitamin on cognitive function: A randomized clinical trial. Alzheimer’s Dement. 2022 Sep 14. doi: 10.1002/alz.12767.

2. Farina N et al. Vitamin E for Alzheimer’s dementia and mild cognitive impairment. Cochrane Database Syst Rev. 2017 Apr 18;4(4):CD002854. doi: 10.1002/14651858.CD002854.pub5.

3. Yu JT et al. Evidence-based prevention of Alzheimer’s disease: Systematic review and meta-analysis of 243 observational prospective studies and 153 randomised controlled trials. J Neurol Neurosurg Psychiatry. 2020 Nov;91(11):1201-9.

4. Del Pozo Cruz B et al. Association of daily step count and intensity with incident dementia in 78,430 adults living in the UK. JAMA Neurol. 2022 Oct 1;79(10):1059-63.

5. Shang X et al. The association between vision impairment and incidence of dementia and cognitive impairment: A systematic review and meta-analysis. Ophthalmology. 2021 Aug;128(8):1135-49.

6. Loughrey DG et al. Association of age-related hearing loss with cognitive function, cognitive impairment, and dementia: A systematic review and meta-analysis. JAMA Otolaryngol Head Neck Surg. 2018 Feb 1;144(2):115-26.

7. Amieva H et al. Self-reported hearing loss, hearing aids, and cognitive decline in elderly adults: A 25-year study. J Am Geriatr Soc. 2015 Oct;63(10):2099-104.

Publications
Topics
Sections

A 67-year-old man asks about what he can do to prevent dementia. He reports his mother had dementia, and he wants to do everything he can to prevent it. Which of the following has evidence of benefit?

A) Thiamine

B) Vitamin E

C) Multivitamin (MV)

D) Keto diet

E) Red wine
 

FDA-approved therapies for dementia

To date the actual therapies for dementia have been disappointing. Donepezil, the most prescribed medication for the treatment of dementia has a number-needed-to treat (NNT) over 17, and causes frequent side effects. Aducanumab was recently approved by the Food and Drug Administration for the treatment of Alzheimer’s disease (AD), but controversy has arisen, as the clinical results were modest, and the price tag will be large – estimated at $30,000-$50,000/year.

Preventive options that may decrease the likelihood of dementia

Patients often ask the question stated above. Regarding how to respond to that question, choice C, MV, has some recent evidence of benefit. Baker and colleagues studied the effect of cocoa extract and multivitamins on cognitive function in the COSMOS-Mind trial.1 A total of 2,262 people were enrolled, and over 90% completed baseline and at least one annual cognitive assessment. Cocoa extract had no impact on global cognition (confidence interval [CI], –.02-.08, P = .28), but MV supplementation did have a statistically significant impact on global cognition (CI, .02-.12, P less than .007).

Paauw_Doug_SEATTLE_2019_web.jpg
Dr. Douglas S. Paauw

Vitamin E has been enthusiastically endorsed in the past as a treatment to prevent cognitive decline. The most recent Cochrane review on vitamin E concluded there was no evidence that the alpha-tocopherol form of vitamin E given to people with MCI prevents progression to dementia, or that it improves cognitive function in people with MCI or dementia due to AD.2

Exercise has long been a mainstay of our advice to patients as something they can do to help prevent dementia. Yu and colleagues did a meta-analysis of almost 400 randomized controlled trials and observational studies to grade the evidence on different interventions.3 They gave exercise a grade B for evidence of benefit.

A recent study addressed this issue, and I think it is helpful on quantifying how much exercise is needed. Del Pozo Cruz and colleagues did a prospective population-based cohort study of 78,000 adults aged 40-79, with an average of 6.9 years of follow up.4 The optimal step count was 9,826 steps (hazard ratio [HR], 0.49; 95% CI, 0.39-0.62) and the minimal step count for benefit was 3,826 steps (HR, 0.75; 95% CI, 0.67-0.83).
 

Modifiable factors

The other major modifiable factors to consider are problems with special senses. Both vision loss and hearing loss have been associated with cognitive impairment.

Shang and colleagues published a meta-analysis of 14 cohort studies addressing vision impairment and cognitive function involving more than 6 million individuals.5 They concluded that vision impairment is associated with an increased risk of both dementia and cognitive impairment in older adults.

Loughrey and colleagues performed a meta-analysis of 36 studies addressing hearing loss and cognitive decline.6 They reported that, among cross-sectional studies, a significant association was found for cognitive impairment (odds ratio [OR], 2.00; 95% CI, 1.39-2.89) and dementia (OR, 2.42; 95% CI, 1.24-4.72). A similar finding was present in prospective cohort studies with a significant association being found for cognitive impairment (OR, 1.22; 95% CI, 1.09-1.36) and dementia (OR, 1.28; 95% CI, 1.02-1.59).

A 25-year prospective, population-based study of patients with hearing loss revealed a difference in the rate of change in MMSE score over the 25-year follow-up between participants with hearing loss not using hearing aids matched with controls who didn’t have hearing loss. Those with untreated hearing loss had more cognitive decline than that of patients without hearing loss.7 The subjects with hearing loss using a hearing aid had no difference in cognitive decline from controls.
 

 

 

Pearl

Several simple and safe interventions may protect our patients from cognitive decline. These include taking a daily multivitamin, walking more than 4,000 steps a day, and optimizing vision and hearing.

Dr. Paauw is professor of medicine in the division of general internal medicine at the University of Washington, Seattle, and he serves as third-year medical student clerkship director at the University of Washington. Contact Dr. Paauw at dpaauw@uw.edu.

References

1. Baker LD et al. Effects of cocoa extract and a multivitamin on cognitive function: A randomized clinical trial. Alzheimer’s Dement. 2022 Sep 14. doi: 10.1002/alz.12767.

2. Farina N et al. Vitamin E for Alzheimer’s dementia and mild cognitive impairment. Cochrane Database Syst Rev. 2017 Apr 18;4(4):CD002854. doi: 10.1002/14651858.CD002854.pub5.

3. Yu JT et al. Evidence-based prevention of Alzheimer’s disease: Systematic review and meta-analysis of 243 observational prospective studies and 153 randomised controlled trials. J Neurol Neurosurg Psychiatry. 2020 Nov;91(11):1201-9.

4. Del Pozo Cruz B et al. Association of daily step count and intensity with incident dementia in 78,430 adults living in the UK. JAMA Neurol. 2022 Oct 1;79(10):1059-63.

5. Shang X et al. The association between vision impairment and incidence of dementia and cognitive impairment: A systematic review and meta-analysis. Ophthalmology. 2021 Aug;128(8):1135-49.

6. Loughrey DG et al. Association of age-related hearing loss with cognitive function, cognitive impairment, and dementia: A systematic review and meta-analysis. JAMA Otolaryngol Head Neck Surg. 2018 Feb 1;144(2):115-26.

7. Amieva H et al. Self-reported hearing loss, hearing aids, and cognitive decline in elderly adults: A 25-year study. J Am Geriatr Soc. 2015 Oct;63(10):2099-104.

A 67-year-old man asks about what he can do to prevent dementia. He reports his mother had dementia, and he wants to do everything he can to prevent it. Which of the following has evidence of benefit?

A) Thiamine

B) Vitamin E

C) Multivitamin (MV)

D) Keto diet

E) Red wine
 

FDA-approved therapies for dementia

To date the actual therapies for dementia have been disappointing. Donepezil, the most prescribed medication for the treatment of dementia has a number-needed-to treat (NNT) over 17, and causes frequent side effects. Aducanumab was recently approved by the Food and Drug Administration for the treatment of Alzheimer’s disease (AD), but controversy has arisen, as the clinical results were modest, and the price tag will be large – estimated at $30,000-$50,000/year.

Preventive options that may decrease the likelihood of dementia

Patients often ask the question stated above. Regarding how to respond to that question, choice C, MV, has some recent evidence of benefit. Baker and colleagues studied the effect of cocoa extract and multivitamins on cognitive function in the COSMOS-Mind trial.1 A total of 2,262 people were enrolled, and over 90% completed baseline and at least one annual cognitive assessment. Cocoa extract had no impact on global cognition (confidence interval [CI], –.02-.08, P = .28), but MV supplementation did have a statistically significant impact on global cognition (CI, .02-.12, P less than .007).

Paauw_Doug_SEATTLE_2019_web.jpg
Dr. Douglas S. Paauw

Vitamin E has been enthusiastically endorsed in the past as a treatment to prevent cognitive decline. The most recent Cochrane review on vitamin E concluded there was no evidence that the alpha-tocopherol form of vitamin E given to people with MCI prevents progression to dementia, or that it improves cognitive function in people with MCI or dementia due to AD.2

Exercise has long been a mainstay of our advice to patients as something they can do to help prevent dementia. Yu and colleagues did a meta-analysis of almost 400 randomized controlled trials and observational studies to grade the evidence on different interventions.3 They gave exercise a grade B for evidence of benefit.

A recent study addressed this issue, and I think it is helpful on quantifying how much exercise is needed. Del Pozo Cruz and colleagues did a prospective population-based cohort study of 78,000 adults aged 40-79, with an average of 6.9 years of follow up.4 The optimal step count was 9,826 steps (hazard ratio [HR], 0.49; 95% CI, 0.39-0.62) and the minimal step count for benefit was 3,826 steps (HR, 0.75; 95% CI, 0.67-0.83).
 

Modifiable factors

The other major modifiable factors to consider are problems with special senses. Both vision loss and hearing loss have been associated with cognitive impairment.

Shang and colleagues published a meta-analysis of 14 cohort studies addressing vision impairment and cognitive function involving more than 6 million individuals.5 They concluded that vision impairment is associated with an increased risk of both dementia and cognitive impairment in older adults.

Loughrey and colleagues performed a meta-analysis of 36 studies addressing hearing loss and cognitive decline.6 They reported that, among cross-sectional studies, a significant association was found for cognitive impairment (odds ratio [OR], 2.00; 95% CI, 1.39-2.89) and dementia (OR, 2.42; 95% CI, 1.24-4.72). A similar finding was present in prospective cohort studies with a significant association being found for cognitive impairment (OR, 1.22; 95% CI, 1.09-1.36) and dementia (OR, 1.28; 95% CI, 1.02-1.59).

A 25-year prospective, population-based study of patients with hearing loss revealed a difference in the rate of change in MMSE score over the 25-year follow-up between participants with hearing loss not using hearing aids matched with controls who didn’t have hearing loss. Those with untreated hearing loss had more cognitive decline than that of patients without hearing loss.7 The subjects with hearing loss using a hearing aid had no difference in cognitive decline from controls.
 

 

 

Pearl

Several simple and safe interventions may protect our patients from cognitive decline. These include taking a daily multivitamin, walking more than 4,000 steps a day, and optimizing vision and hearing.

Dr. Paauw is professor of medicine in the division of general internal medicine at the University of Washington, Seattle, and he serves as third-year medical student clerkship director at the University of Washington. Contact Dr. Paauw at dpaauw@uw.edu.

References

1. Baker LD et al. Effects of cocoa extract and a multivitamin on cognitive function: A randomized clinical trial. Alzheimer’s Dement. 2022 Sep 14. doi: 10.1002/alz.12767.

2. Farina N et al. Vitamin E for Alzheimer’s dementia and mild cognitive impairment. Cochrane Database Syst Rev. 2017 Apr 18;4(4):CD002854. doi: 10.1002/14651858.CD002854.pub5.

3. Yu JT et al. Evidence-based prevention of Alzheimer’s disease: Systematic review and meta-analysis of 243 observational prospective studies and 153 randomised controlled trials. J Neurol Neurosurg Psychiatry. 2020 Nov;91(11):1201-9.

4. Del Pozo Cruz B et al. Association of daily step count and intensity with incident dementia in 78,430 adults living in the UK. JAMA Neurol. 2022 Oct 1;79(10):1059-63.

5. Shang X et al. The association between vision impairment and incidence of dementia and cognitive impairment: A systematic review and meta-analysis. Ophthalmology. 2021 Aug;128(8):1135-49.

6. Loughrey DG et al. Association of age-related hearing loss with cognitive function, cognitive impairment, and dementia: A systematic review and meta-analysis. JAMA Otolaryngol Head Neck Surg. 2018 Feb 1;144(2):115-26.

7. Amieva H et al. Self-reported hearing loss, hearing aids, and cognitive decline in elderly adults: A 25-year study. J Am Geriatr Soc. 2015 Oct;63(10):2099-104.

Publications
Publications
Topics
Article Type
Sections
Teambase XML
<?xml version="1.0" encoding="UTF-8"?>
<!--$RCSfile: InCopy_agile.xsl,v $ $Revision: 1.35 $-->
<!--$RCSfile: drupal.xsl,v $ $Revision: 1.7 $-->
<root generator="drupal.xsl" gversion="1.7"> <header> <fileName>160696</fileName> <TBEID>0C046221.SIG</TBEID> <TBUniqueIdentifier>MD_0C046221</TBUniqueIdentifier> <newsOrJournal>News</newsOrJournal> <publisherName>Frontline Medical Communications</publisherName> <storyname>How can I keep from losing my mi</storyname> <articleType>353</articleType> <TBLocation>QC Done-All Pubs</TBLocation> <QCDate>20221026T084556</QCDate> <firstPublished>20221026T091829</firstPublished> <LastPublished>20221026T091829</LastPublished> <pubStatus qcode="stat:"/> <embargoDate/> <killDate/> <CMSDate>20221026T091829</CMSDate> <articleSource/> <facebookInfo/> <meetingNumber/> <byline>Douglas S. Paauw</byline> <bylineText>DOUGLAS S. PAAUW, MD</bylineText> <bylineFull>DOUGLAS S. PAAUW, MD</bylineFull> <bylineTitleText/> <USOrGlobal/> <wireDocType/> <newsDocType>Column</newsDocType> <journalDocType/> <linkLabel/> <pageRange/> <citation/> <quizID/> <indexIssueDate/> <itemClass qcode="ninat:text"/> <provider qcode="provider:imng"> <name>IMNG Medical Media</name> <rightsInfo> <copyrightHolder> <name>Frontline Medical News</name> </copyrightHolder> <copyrightNotice>Copyright (c) 2015 Frontline Medical News, a Frontline Medical Communications Inc. company. All rights reserved. This material may not be published, broadcast, copied, or otherwise reproduced or distributed without the prior written permission of Frontline Medical Communications Inc.</copyrightNotice> </rightsInfo> </provider> <abstract/> <metaDescription>A 67-year-old man asks about what he can do to prevent dementia. He reports his mother had dementia, and he wants to do everything he can to prevent it. Which o</metaDescription> <articlePDF/> <teaserImage>248215</teaserImage> <teaser>“To date the actual therapies for dementia have been disappointing.”</teaser> <title>How can I keep from losing my mind?</title> <deck/> <disclaimer/> <AuthorList/> <articleURL/> <doi/> <pubMedID/> <publishXMLStatus/> <publishXMLVersion>1</publishXMLVersion> <useEISSN>0</useEISSN> <urgency/> <pubPubdateYear/> <pubPubdateMonth/> <pubPubdateDay/> <pubVolume/> <pubNumber/> <wireChannels/> <primaryCMSID/> <CMSIDs/> <keywords/> <seeAlsos/> <publications_g> <publicationData> <publicationCode>fp</publicationCode> <pubIssueName/> <pubArticleType/> <pubTopics/> <pubCategories/> <pubSections/> </publicationData> <publicationData> <publicationCode>im</publicationCode> <pubIssueName/> <pubArticleType/> <pubTopics/> <pubCategories/> <pubSections/> </publicationData> <publicationData> <publicationCode>cpn</publicationCode> <pubIssueName/> <pubArticleType/> <pubTopics/> <pubCategories/> <pubSections/> </publicationData> <publicationData> <publicationCode>nr</publicationCode> <pubIssueName/> <pubArticleType/> <pubTopics/> <pubCategories/> <pubSections/> <journalTitle>Neurology Reviews</journalTitle> <journalFullTitle>Neurology Reviews</journalFullTitle> <copyrightStatement>2018 Frontline Medical Communications Inc.,</copyrightStatement> </publicationData> </publications_g> <publications> <term>15</term> <term canonical="true">21</term> <term>9</term> <term>22</term> </publications> <sections> <term canonical="true">39786</term> <term>41022</term> <term>52</term> </sections> <topics> <term>248</term> <term canonical="true">280</term> <term>180</term> <term>215</term> <term>258</term> </topics> <links> <link> <itemClass qcode="ninat:picture"/> <altRep contenttype="image/jpeg">images/2400c6a0.jpg</altRep> <description role="drol:caption">Dr. Douglas S. Paauw</description> <description role="drol:credit"/> </link> </links> </header> <itemSet> <newsItem> <itemMeta> <itemRole>Main</itemRole> <itemClass>text</itemClass> <title>How can I keep from losing my mind?</title> <deck/> </itemMeta> <itemContent> <p> <span class="tag metaDescription">A 67-year-old man asks about what he can do to prevent dementia. He reports his mother had dementia, and he wants to do everything he can to prevent it. Which of the following has evidence of benefit?</span> </p> <p>A) Thiamine<br/><br/>B) Vitamin E<br/><br/>C) Multivitamin injection (MVI)<br/><br/>D) Keto diet<br/><br/>E) Red wine<br/><br/></p> <h2>FDA-approved therapies for dementia</h2> <p>To date the actual therapies for dementia have been disappointing. Donepezil, the most prescribed medication for the treatment of dementia has a number-needed-to treat (NNT) over 17, and causes frequent side effects. Aducanumab was recently approved by the Food and Drug Administration for the treatment of Alzheimer’s disease (AD), but controversy has arisen, as the clinical results were modest, and the price tag will be large – estimated at $30,000-$50,000/year.</p> <h2>Preventive options that may decrease the likelihood of dementia</h2> <p>Patients often ask the question stated above. Regarding how to respond to that question, choice C, MVI, has some recent evidence of benefit. Baker and colleagues studied the effect of cocoa extract and multivitamins on cognitive function in the COSMOS-Mind trial.<sup>1</sup> A total of 2,262 people were enrolled, and over 90% completed baseline and at least one annual cognitive assessment. Cocoa extract had no impact on global cognition (confidence interval [CI], –.02-.08, <em>P</em> = .28), but MVI supplementation did have a statistically significant impact on global cognition (CI, .02-.12, <em>P</em> less than .007). </p> <p>[[{"fid":"248215","view_mode":"medstat_image_flush_right","fields":{"format":"medstat_image_flush_right","field_file_image_alt_text[und][0][value]":"Dr. Douglas S. Paauw, University of Washington, Seattle","field_file_image_credit[und][0][value]":"","field_file_image_caption[und][0][value]":"Dr. Douglas S. Paauw"},"type":"media","attributes":{"class":"media-element file-medstat_image_flush_right"}}]]Vitamin E has been enthusiastically endorsed in the past as a treatment to prevent cognitive decline. The most recent Cochrane review on vitamin E concluded there was no evidence that the alpha-tocopherol form of vitamin E given to people with MCI prevents progression to dementia, or that it improves cognitive function in people with MCI or dementia due to AD.<sup>2</sup><br/><br/>Exercise has long been a mainstay of our advice to patients as something they can do to help prevent dementia. Yu and colleagues did a meta-analysis of almost 400 randomized controlled trials and observational studies to grade the evidence on different interventions.<sup>3</sup> They gave exercise a grade B for evidence of benefit. <br/><br/>A recent study addressed this issue, and I think it is helpful on quantifying how much exercise is needed. Del Pozo Cruz and colleagues did a prospective population-based cohort study of 78,000 adults aged 40-79, with an average of 6.9 years of follow up.<sup>4</sup> The optimal step count was 9,826 steps (hazard ratio [HR], 0.49; 95% CI, 0.39-0.62) and the minimal step count for benefit was 3,826 steps (HR, 0.75; 95% CI, 0.67-0.83).<br/><br/></p> <h2>Modifiable factors</h2> <p>The other major modifiable factors to consider are problems with special senses. Both vision loss and hearing loss have been associated with cognitive impairment. </p> <p>Shang and colleagues published a meta-analysis of 14 cohort studies addressing vision impairment and cognitive function involving more than 6 million individuals.<sup>5</sup> They concluded that vision impairment is associated with an increased risk of both dementia and cognitive impairment in older adults.<br/><br/>Loughrey and colleagues performed a meta-analysis of 36 studies addressing hearing loss and cognitive decline.<sup>6</sup> They reported that, among cross-sectional studies, a significant association was found for cognitive impairment (odds ratio [OR], 2.00; 95% CI, 1.39-2.89) and dementia (OR, 2.42; 95% CI, 1.24-4.72). A similar finding was present in prospective cohort studies with a significant association being found for cognitive impairment (OR, 1.22; 95% CI, 1.09-1.36) and dementia (OR, 1.28; 95% CI, 1.02-1.59). <br/><br/>A 25-year prospective, population-based study of patients with hearing loss revealed a difference in the rate of change in MMSE score over the 25-year follow-up between participants with hearing loss not using hearing aids matched with controls who didn’t have hearing loss. Those with untreated hearing loss had more cognitive decline than that of patients without hearing loss.<sup>7</sup> The subjects with hearing loss using a hearing aid had no difference in cognitive decline from controls.<br/><br/></p> <h2>Pearl</h2> <p>Several simple and safe interventions may protect our patients from cognitive decline. These include taking a daily multivitamin, walking more than 4,000 steps a day, and optimizing vision and hearing.</p> <p> <em>Dr. Paauw is professor of medicine in the division of general internal medicine at the University of Washington, Seattle, and he serves as third-year medical student clerkship director at the University of Washington. Contact Dr. Paauw at <span class="Hyperlink"><a href="mailto:dpaauw%40uw.edu?subject=">dpaauw@uw.edu</a></span>.</em> </p> <h2>References</h2> <p>1. Baker LD et al. Effects of cocoa extract and a multivitamin on cognitive function: A randomized clinical trial. A<span class="Hyperlink"><a href="https://pubmed.ncbi.nlm.nih.gov/36102337/">lzheimer’s Dement. 2022 Sep 14</a></span>. doi: 10.1002/alz.12767.<br/><br/>2. Farina N et al. Vitamin E for Alzheimer’s dementia and mild cognitive impairment. <span class="Hyperlink"><a href="https://pubmed.ncbi.nlm.nih.gov/28418065/">Cochrane Database Syst Rev. 2017 Apr 18;4(4)</a></span>:CD002854. doi: 10.1002/14651858.CD002854.pub5.<br/><br/>3. Yu JT et al. Evidence-based prevention of Alzheimer’s disease: Systematic review and meta-analysis of 243 observational prospective studies and 153 randomised controlled trials. <span class="Hyperlink"><a href="https://pubmed.ncbi.nlm.nih.gov/32690803/">J Neurol Neurosurg Psychiatry. 2020 Nov;91(11):1201-9</a></span>. <br/><br/>4. Del Pozo Cruz B et al. Association of daily step count and intensity with incident dementia in 78,430 adults living in the UK. <span class="Hyperlink"><a href="https://pubmed.ncbi.nlm.nih.gov/36066874/">JAMA Neurol. 2022 Oct 1;79(10):1059-63</a></span>.<br/><br/>5. Shang X et al. The association between vision impairment and incidence of dementia and cognitive impairment: A systematic review and meta-analysis. <span class="Hyperlink"><a href="https://pubmed.ncbi.nlm.nih.gov/33422559/">Ophthalmology. 2021 Aug;128(8):1135-49</a></span>.<br/><br/>6. Loughrey DG et al. Association of age-related hearing loss with cognitive function, cognitive impairment, and dementia: A systematic review and meta-analysis. <span class="Hyperlink"><a href="https://pubmed.ncbi.nlm.nih.gov/29222544/">JAMA Otolaryngol Head Neck Surg. 2018 Feb 1;144(2):115-26</a></span>.<br/><br/>7. Amieva H et al. Self-reported hearing loss, hearing aids, and cognitive decline in elderly adults: A 25-year study. <span class="Hyperlink"><a href="https://pubmed.ncbi.nlm.nih.gov/26480972/">J Am Geriatr Soc. 2015 Oct;63(10):2099-104</a></span>.</p> </itemContent> </newsItem> <newsItem> <itemMeta> <itemRole>teaser</itemRole> <itemClass>text</itemClass> <title/> <deck/> </itemMeta> <itemContent> </itemContent> </newsItem> </itemSet></root>
Disallow All Ads
Content Gating
No Gating (article Unlocked/Free)
Alternative CME
Disqus Comments
Default
Use ProPublica
Hide sidebar & use full width
render the right sidebar.
Conference Recap Checkbox
Not Conference Recap
Clinical Edge
Display the Slideshow in this Article
Medscape Article
Display survey writer
Reuters content
Disable Inline Native ads
WebMD Article

Burnout and stress of today: How do we cope?

Article Type
Changed
Wed, 08/03/2022 - 16:44

 

As I prepared to write my monthly column, I came across the statistic that 23% of physicians and 40% of nurses plan to leave their practices in the next 2 years.1

Interestingly, the group that seems to be least impacted by this was health care administrators (with 12% of them planning on leaving their jobs).

I couldn’t stop thinking about these percentages.

Paauw_Doug_SEATTLE_2019_web.jpg
Dr. Douglas S. Paauw

I am reminded every day of the commitment and excellence of my colleagues in the health care field, and I do not want to lose them. I am hoping the following information and my thoughts on this topic will be helpful for those thinking about leaving health care.
 

Surgeon general’s burnout report

The surgeon general recently released a report on addressing health care worker burnout.2 It includes several very interesting and appropriate observations. I will summarize the most important ones here:

1. Our health depends on the well-being of our health workforce.

2. Direct harm to health care workers can lead to anxiety, depression, insomnia, and interpersonal and relationship struggles.

3. Health care workers experience exhaustion from providing overwhelming care and empathy.

4. Health care workers spend less time with patients and too much time with EHRs.

5. There are health workforce shortages.

The report is comprehensive, and everything in it is correct. The real issue is how does it go from being a report to true actionable items that we as health care professionals benefit from? I think in regards to exhaustion from overwhelming care responsibilities, and empathy fatigue, we need better boundaries.

Those who go into medicine, and especially those who go into primary care, always put the patients’ needs first. When operating in a broken system, it stays broken when individuals cover for the deficiencies in the system. Adding four extra patients every day because there is no one to refer them to with availability is injurious to the health care provider, and those providers who accept these additional patients will eventually be part of the 23% who want to leave their jobs. It feels awful to say no, but until the system stops accommodating there will not be substantial change.
 

The empathy drain

One of the unreported stresses of open access for patients through EHR communications is the empathy drain on physicians. When I see a patient in clinic with chronic symptoms or issues, I spend important time making sure we have a plan and an agreed upon time frame.

With the EHR, patients frequently send multiple messages for the same symptoms between visits. It is okay to redirect the patient and share that these issues will be discussed at length at appointments. My reasoning on this is that I think it is better for me to better care for myself and stay as the doctor for my patients, than always say yes to limitless needs and soon be looking for the off ramp.

The following statistic in the surgeon general’s report really hit home. For every hour of direct patient care, physicians currently spend 2 hours on the EHR system. Most practices allow 10%-20% of time for catch up, where with statistics like this it should be 50%. This concept is fully lost on administrators, or ignored.

It is only when we refuse to continue to accept and follow a broken system that it will change. A minority of internal medicine and family doctors (4.5% in 2018) practice in direct primary care models, where these issues are addressed. Unfortunately, this model as it is currently available is not an option for lower income patients.

A major theme in the surgeon general’s report was that administrative burdens need to be reduced by 75% by 2025. When I look at the report, I see the suggestions, I just don’t see how it will be achieved. Despite almost all clinics moving to the EHR, paperwork in the form of faxes and forms has increased.

A sweeping reform would be needed to eliminate daily faxes from PT offices, visiting nurse services, prior authorization, patients reminders from insurance companies, and disability forms from patients. I am glad that there is acknowledgment of the problem, but this change will take more than 3 years.
 

 

 

Takeaways

So what do we do?

Be good to yourself, and your colleagues. The pandemic has isolated us, which accelerates burnout.

Reach out to people you care about.

We are all feeling this. Set boundaries that allow you to care for yourself, and accept that you are doing your best, even if you can’t meet the needs of all your patients all the time.
 

Dr. Paauw is professor of medicine in the division of general internal medicine at the University of Washington, Seattle, and he serves as third-year medical student clerkship director at the University of Washington. He is a member of the editorial advisory board of Internal Medicine News. Dr. Paauw has no conflicts to disclose. Contact him at imnews@mdedge.com.

References

1. Sinsky CA et al. Covid-related stress and work intentions in a sample of US health care workers. Mayo Clin Proc Innov Qual Outcomes. 2021 Dec;5(6):1165-73.

2. Addressing health worker burnout. The U.S. Surgeon General’s advisory on building a thriving health workforce.

Publications
Topics
Sections

 

As I prepared to write my monthly column, I came across the statistic that 23% of physicians and 40% of nurses plan to leave their practices in the next 2 years.1

Interestingly, the group that seems to be least impacted by this was health care administrators (with 12% of them planning on leaving their jobs).

I couldn’t stop thinking about these percentages.

Paauw_Doug_SEATTLE_2019_web.jpg
Dr. Douglas S. Paauw

I am reminded every day of the commitment and excellence of my colleagues in the health care field, and I do not want to lose them. I am hoping the following information and my thoughts on this topic will be helpful for those thinking about leaving health care.
 

Surgeon general’s burnout report

The surgeon general recently released a report on addressing health care worker burnout.2 It includes several very interesting and appropriate observations. I will summarize the most important ones here:

1. Our health depends on the well-being of our health workforce.

2. Direct harm to health care workers can lead to anxiety, depression, insomnia, and interpersonal and relationship struggles.

3. Health care workers experience exhaustion from providing overwhelming care and empathy.

4. Health care workers spend less time with patients and too much time with EHRs.

5. There are health workforce shortages.

The report is comprehensive, and everything in it is correct. The real issue is how does it go from being a report to true actionable items that we as health care professionals benefit from? I think in regards to exhaustion from overwhelming care responsibilities, and empathy fatigue, we need better boundaries.

Those who go into medicine, and especially those who go into primary care, always put the patients’ needs first. When operating in a broken system, it stays broken when individuals cover for the deficiencies in the system. Adding four extra patients every day because there is no one to refer them to with availability is injurious to the health care provider, and those providers who accept these additional patients will eventually be part of the 23% who want to leave their jobs. It feels awful to say no, but until the system stops accommodating there will not be substantial change.
 

The empathy drain

One of the unreported stresses of open access for patients through EHR communications is the empathy drain on physicians. When I see a patient in clinic with chronic symptoms or issues, I spend important time making sure we have a plan and an agreed upon time frame.

With the EHR, patients frequently send multiple messages for the same symptoms between visits. It is okay to redirect the patient and share that these issues will be discussed at length at appointments. My reasoning on this is that I think it is better for me to better care for myself and stay as the doctor for my patients, than always say yes to limitless needs and soon be looking for the off ramp.

The following statistic in the surgeon general’s report really hit home. For every hour of direct patient care, physicians currently spend 2 hours on the EHR system. Most practices allow 10%-20% of time for catch up, where with statistics like this it should be 50%. This concept is fully lost on administrators, or ignored.

It is only when we refuse to continue to accept and follow a broken system that it will change. A minority of internal medicine and family doctors (4.5% in 2018) practice in direct primary care models, where these issues are addressed. Unfortunately, this model as it is currently available is not an option for lower income patients.

A major theme in the surgeon general’s report was that administrative burdens need to be reduced by 75% by 2025. When I look at the report, I see the suggestions, I just don’t see how it will be achieved. Despite almost all clinics moving to the EHR, paperwork in the form of faxes and forms has increased.

A sweeping reform would be needed to eliminate daily faxes from PT offices, visiting nurse services, prior authorization, patients reminders from insurance companies, and disability forms from patients. I am glad that there is acknowledgment of the problem, but this change will take more than 3 years.
 

 

 

Takeaways

So what do we do?

Be good to yourself, and your colleagues. The pandemic has isolated us, which accelerates burnout.

Reach out to people you care about.

We are all feeling this. Set boundaries that allow you to care for yourself, and accept that you are doing your best, even if you can’t meet the needs of all your patients all the time.
 

Dr. Paauw is professor of medicine in the division of general internal medicine at the University of Washington, Seattle, and he serves as third-year medical student clerkship director at the University of Washington. He is a member of the editorial advisory board of Internal Medicine News. Dr. Paauw has no conflicts to disclose. Contact him at imnews@mdedge.com.

References

1. Sinsky CA et al. Covid-related stress and work intentions in a sample of US health care workers. Mayo Clin Proc Innov Qual Outcomes. 2021 Dec;5(6):1165-73.

2. Addressing health worker burnout. The U.S. Surgeon General’s advisory on building a thriving health workforce.

 

As I prepared to write my monthly column, I came across the statistic that 23% of physicians and 40% of nurses plan to leave their practices in the next 2 years.1

Interestingly, the group that seems to be least impacted by this was health care administrators (with 12% of them planning on leaving their jobs).

I couldn’t stop thinking about these percentages.

Paauw_Doug_SEATTLE_2019_web.jpg
Dr. Douglas S. Paauw

I am reminded every day of the commitment and excellence of my colleagues in the health care field, and I do not want to lose them. I am hoping the following information and my thoughts on this topic will be helpful for those thinking about leaving health care.
 

Surgeon general’s burnout report

The surgeon general recently released a report on addressing health care worker burnout.2 It includes several very interesting and appropriate observations. I will summarize the most important ones here:

1. Our health depends on the well-being of our health workforce.

2. Direct harm to health care workers can lead to anxiety, depression, insomnia, and interpersonal and relationship struggles.

3. Health care workers experience exhaustion from providing overwhelming care and empathy.

4. Health care workers spend less time with patients and too much time with EHRs.

5. There are health workforce shortages.

The report is comprehensive, and everything in it is correct. The real issue is how does it go from being a report to true actionable items that we as health care professionals benefit from? I think in regards to exhaustion from overwhelming care responsibilities, and empathy fatigue, we need better boundaries.

Those who go into medicine, and especially those who go into primary care, always put the patients’ needs first. When operating in a broken system, it stays broken when individuals cover for the deficiencies in the system. Adding four extra patients every day because there is no one to refer them to with availability is injurious to the health care provider, and those providers who accept these additional patients will eventually be part of the 23% who want to leave their jobs. It feels awful to say no, but until the system stops accommodating there will not be substantial change.
 

The empathy drain

One of the unreported stresses of open access for patients through EHR communications is the empathy drain on physicians. When I see a patient in clinic with chronic symptoms or issues, I spend important time making sure we have a plan and an agreed upon time frame.

With the EHR, patients frequently send multiple messages for the same symptoms between visits. It is okay to redirect the patient and share that these issues will be discussed at length at appointments. My reasoning on this is that I think it is better for me to better care for myself and stay as the doctor for my patients, than always say yes to limitless needs and soon be looking for the off ramp.

The following statistic in the surgeon general’s report really hit home. For every hour of direct patient care, physicians currently spend 2 hours on the EHR system. Most practices allow 10%-20% of time for catch up, where with statistics like this it should be 50%. This concept is fully lost on administrators, or ignored.

It is only when we refuse to continue to accept and follow a broken system that it will change. A minority of internal medicine and family doctors (4.5% in 2018) practice in direct primary care models, where these issues are addressed. Unfortunately, this model as it is currently available is not an option for lower income patients.

A major theme in the surgeon general’s report was that administrative burdens need to be reduced by 75% by 2025. When I look at the report, I see the suggestions, I just don’t see how it will be achieved. Despite almost all clinics moving to the EHR, paperwork in the form of faxes and forms has increased.

A sweeping reform would be needed to eliminate daily faxes from PT offices, visiting nurse services, prior authorization, patients reminders from insurance companies, and disability forms from patients. I am glad that there is acknowledgment of the problem, but this change will take more than 3 years.
 

 

 

Takeaways

So what do we do?

Be good to yourself, and your colleagues. The pandemic has isolated us, which accelerates burnout.

Reach out to people you care about.

We are all feeling this. Set boundaries that allow you to care for yourself, and accept that you are doing your best, even if you can’t meet the needs of all your patients all the time.
 

Dr. Paauw is professor of medicine in the division of general internal medicine at the University of Washington, Seattle, and he serves as third-year medical student clerkship director at the University of Washington. He is a member of the editorial advisory board of Internal Medicine News. Dr. Paauw has no conflicts to disclose. Contact him at imnews@mdedge.com.

References

1. Sinsky CA et al. Covid-related stress and work intentions in a sample of US health care workers. Mayo Clin Proc Innov Qual Outcomes. 2021 Dec;5(6):1165-73.

2. Addressing health worker burnout. The U.S. Surgeon General’s advisory on building a thriving health workforce.

Publications
Publications
Topics
Article Type
Sections
Teambase XML
<?xml version="1.0" encoding="UTF-8"?>
<!--$RCSfile: InCopy_agile.xsl,v $ $Revision: 1.35 $-->
<!--$RCSfile: drupal.xsl,v $ $Revision: 1.7 $-->
<root generator="drupal.xsl" gversion="1.7"> <header> <fileName>159295</fileName> <TBEID>0C04445B.SIG</TBEID> <TBUniqueIdentifier>MD_0C04445B</TBUniqueIdentifier> <newsOrJournal>News</newsOrJournal> <publisherName>Frontline Medical Communications</publisherName> <storyname>Burnout and Stress of Today: How</storyname> <articleType>353</articleType> <TBLocation>QC Done-All Pubs</TBLocation> <QCDate>20220803T134531</QCDate> <firstPublished>20220803T134745</firstPublished> <LastPublished>20220803T134745</LastPublished> <pubStatus qcode="stat:"/> <embargoDate/> <killDate/> <CMSDate>20220803T134745</CMSDate> <articleSource/> <facebookInfo/> <meetingNumber/> <byline>Douglas S. Paauw</byline> <bylineText>DOUGLAS S. PAAUW, MD</bylineText> <bylineFull>DOUGLAS S. PAAUW, MD</bylineFull> <bylineTitleText/> <USOrGlobal/> <wireDocType/> <newsDocType>Opinion</newsDocType> <journalDocType/> <linkLabel/> <pageRange/> <citation/> <quizID/> <indexIssueDate/> <itemClass qcode="ninat:text"/> <provider qcode="provider:imng"> <name>IMNG Medical Media</name> <rightsInfo> <copyrightHolder> <name>Frontline Medical News</name> </copyrightHolder> <copyrightNotice>Copyright (c) 2015 Frontline Medical News, a Frontline Medical Communications Inc. company. All rights reserved. This material may not be published, broadcast, copied, or otherwise reproduced or distributed without the prior written permission of Frontline Medical Communications Inc.</copyrightNotice> </rightsInfo> </provider> <abstract/> <metaDescription>As I prepared to write my monthly column, I came across the statistic that 23% of physicians and 40% of nurses plan to leave their practices in the next 2 years</metaDescription> <articlePDF/> <teaserImage>248215</teaserImage> <teaser>“I am reminded every day of the commitment and excellence of my colleagues in the health care field, and I do not want to lose them.”</teaser> <title>Burnout and stress of today: How do we cope?</title> <deck/> <disclaimer/> <AuthorList/> <articleURL/> <doi/> <pubMedID/> <publishXMLStatus/> <publishXMLVersion>1</publishXMLVersion> <useEISSN>0</useEISSN> <urgency/> <pubPubdateYear/> <pubPubdateMonth/> <pubPubdateDay/> <pubVolume/> <pubNumber/> <wireChannels/> <primaryCMSID/> <CMSIDs/> <keywords/> <seeAlsos/> <publications_g> <publicationData> <publicationCode>im</publicationCode> <pubIssueName/> <pubArticleType/> <pubTopics/> <pubCategories/> <pubSections/> </publicationData> <publicationData> <publicationCode>fp</publicationCode> <pubIssueName/> <pubArticleType/> <pubTopics/> <pubCategories/> <pubSections/> </publicationData> <publicationData> <publicationCode>cpn</publicationCode> <pubIssueName/> <pubArticleType/> <pubTopics/> <pubCategories/> <pubSections/> </publicationData> <publicationData> <publicationCode>chph</publicationCode> <pubIssueName/> <pubArticleType/> <pubTopics/> <pubCategories/> <pubSections/> </publicationData> <publicationData> <publicationCode>endo</publicationCode> <pubIssueName/> <pubArticleType/> <pubTopics/> <pubCategories/> <pubSections/> </publicationData> <publicationData> <publicationCode>card</publicationCode> <pubIssueName/> <pubArticleType/> <pubTopics/> <pubCategories/> <pubSections/> </publicationData> <publicationData> <publicationCode>hemn</publicationCode> <pubIssueName/> <pubArticleType/> <pubTopics/> <pubCategories/> <pubSections/> </publicationData> <publicationData> <publicationCode>idprac</publicationCode> <pubIssueName/> <pubArticleType/> <pubTopics/> <pubCategories/> <pubSections/> </publicationData> <publicationData> <publicationCode>rn</publicationCode> <pubIssueName/> <pubArticleType/> <pubTopics/> <pubCategories/> <pubSections/> </publicationData> <publicationData> <publicationCode>ob</publicationCode> <pubIssueName/> <pubArticleType/> <pubTopics/> <pubCategories/> <pubSections/> </publicationData> <publicationData> <publicationCode>oncr</publicationCode> <pubIssueName/> <pubArticleType/> <pubTopics/> <pubCategories/> <pubSections/> </publicationData> <publicationData> <publicationCode>pn</publicationCode> <pubIssueName/> <pubArticleType/> <pubTopics/> <pubCategories/> <pubSections/> </publicationData> <publicationData> <publicationCode>mdsurg</publicationCode> <pubIssueName/> <pubArticleType/> <pubTopics/> <pubCategories/> <pubSections/> <journalTitle/> <journalFullTitle/> <copyrightStatement>2018 Frontline Medical Communications Inc.,</copyrightStatement> </publicationData> <publicationData> <publicationCode>nr</publicationCode> <pubIssueName/> <pubArticleType/> <pubTopics/> <pubCategories/> <pubSections/> <journalTitle>Neurology Reviews</journalTitle> <journalFullTitle>Neurology Reviews</journalFullTitle> <copyrightStatement>2018 Frontline Medical Communications Inc.,</copyrightStatement> </publicationData> <publicationData> <publicationCode>skin</publicationCode> <pubIssueName/> <pubArticleType/> <pubTopics/> <pubCategories/> <pubSections/> </publicationData> </publications_g> <publications> <term canonical="true">21</term> <term>15</term> <term>9</term> <term>6</term> <term>34</term> <term>5</term> <term>18</term> <term>20</term> <term>26</term> <term>23</term> <term>31</term> <term>25</term> <term>52226</term> <term>22</term> <term>13</term> </publications> <sections> <term canonical="true">39786</term> <term>41022</term> <term>52</term> </sections> <topics> <term>38029</term> <term canonical="true">248</term> <term>202</term> <term>184</term> <term>278</term> </topics> <links> <link> <itemClass qcode="ninat:picture"/> <altRep contenttype="image/jpeg">images/2400c6a0.jpg</altRep> <description role="drol:caption">Dr. Douglas S. Paauw</description> <description role="drol:credit"/> </link> </links> </header> <itemSet> <newsItem> <itemMeta> <itemRole>Main</itemRole> <itemClass>text</itemClass> <title>Burnout and stress of today: How do we cope?</title> <deck/> </itemMeta> <itemContent> <p> <span class="tag metaDescription">As I prepared to write my monthly column, I came across the statistic that 23% of physicians and 40% of nurses plan to leave their practices in the next 2 years.<sup>1</sup> </span> </p> <p>Interestingly, the group that seems to be least impacted by this was health care administrators (with 12% of them planning on leaving their jobs).<br/><br/>I couldn’t stop thinking about these percentages.<br/><br/>[[{"fid":"248215","view_mode":"medstat_image_flush_right","fields":{"format":"medstat_image_flush_right","field_file_image_alt_text[und][0][value]":"Dr. Douglas S. Paauw, University of Washington, Seattle","field_file_image_credit[und][0][value]":"","field_file_image_caption[und][0][value]":"Dr. Douglas S. Paauw"},"type":"media","attributes":{"class":"media-element file-medstat_image_flush_right"}}]]I am reminded every day of the commitment and excellence of my colleagues in the health care field, and I do not want to lose them. I am hoping the following information and my thoughts on this topic will be helpful for those thinking about leaving health care.<br/><br/></p> <h2>Surgeon general’s burnout report</h2> <p>The surgeon general recently released a report on addressing health care worker burnout.<sup>2</sup> It includes several very interesting and appropriate observations. I will summarize the most important ones here:</p> <p>1. Our health depends on the well-being of our health workforce.<br/><br/>2. Direct harm to health care workers can lead to anxiety, depression, insomnia, and interpersonal and relationship struggles.<br/><br/>3. Health care workers experience exhaustion from providing overwhelming care and empathy.<br/><br/>4. Health care workers spend less time with patients and too much time with EHRs.<br/><br/>5. There are health workforce shortages.<br/><br/>The report is comprehensive, and everything in it is correct. The real issue is how does it go from being a report to true actionable items that we as health care professionals benefit from? I think in regards to exhaustion from overwhelming care responsibilities, and empathy fatigue, we need better boundaries.<br/><br/>Those who go into medicine, and especially those who go into primary care, always put the patients’ needs first. When operating in a broken system, it stays broken when individuals cover for the deficiencies in the system. Adding four extra patients every day because there is no one to refer them to with availability is injurious to the health care provider, and those providers who accept these additional patients will eventually be part of the 23% who want to leave their jobs. It feels awful to say no, but until the system stops accommodating there will not be substantial change.<br/><br/></p> <h2>The empathy drain </h2> <p>One of the unreported stresses of open access for patients through EHR communications is the empathy drain on physicians. When I see a patient in clinic with chronic symptoms or issues, I spend important time making sure we have a plan and an agreed upon time frame.</p> <p>With the EHR, patients frequently send multiple messages for the same symptoms between visits. It is okay to redirect the patient and share that these issues will be discussed at length at appointments. My reasoning on this is that I think it is better for me to better care for myself and stay as the doctor for my patients, than always say yes to limitless needs and soon be looking for the off ramp.<br/><br/>The following statistic in the surgeon general’s report really hit home. For every hour of direct patient care, physicians currently spend 2 hours on the EHR system. Most practices allow 10%-20% of time for catch up, where with statistics like this it should be 50%. This concept is fully lost on administrators, or ignored.<br/><br/>It is only when we refuse to continue to accept and follow a broken system that it will change. A minority of internal medicine and family doctors (4.5% in 2018) practice in direct primary care models, where these issues are addressed. Unfortunately, this model as it is currently available is not an option for lower income patients.<br/><br/>A major theme in the surgeon general’s report was that administrative burdens need to be reduced by 75% by 2025. When I look at the report, I see the suggestions, I just don’t see how it will be achieved. Despite almost all clinics moving to the EHR, paperwork in the form of faxes and forms has increased. <br/><br/>A sweeping reform would be needed to eliminate daily faxes from PT offices, visiting nurse services, prior authorization, patients reminders from insurance companies, and disability forms from patients. I am glad that there is acknowledgment of the problem, but this change will take more than 3 years.<br/><br/></p> <h2>Takeaways</h2> <p>So what do we do? </p> <p>Be good to yourself, and your colleagues. The pandemic has isolated us, which accelerates burnout. <br/><br/>Reach out to people you care about. <br/><br/>We are all feeling this. Set boundaries that allow you to care for yourself, and accept that you are doing your best, even if you can’t meet the needs of all your patients all the time.<br/><br/></p> <p> <em>Dr. Paauw is professor of medicine in the division of general internal medicine at the University of Washington, Seattle, and he serves as third-year medical student clerkship director at the University of Washington. He is a member of the editorial advisory board of Internal Medicine News. Dr. Paauw has no conflicts to disclose. Contact him at <span class="Hyperlink"><a href="mailto:imnews%40mdedge.com?subject=">imnews@mdedge.com</a></span>.</em> </p> <h2>References</h2> <p>1. Sinsky CA et al. Covid-related stress and work intentions in a sample of US health care workers. Mayo Clin Proc Innov Qual Outcomes. <span class="Hyperlink"><a href="https://www.mcpiqojournal.org/article/S2542-4548(21)00126-0/fulltext">2021 Dec;5(6):1165-73</a></span>. <br/><br/>2. <span class="Hyperlink"><a href="http://Surgeongeneral.gov/burnout">Addressing health worker burnout</a></span>. The U.S. Surgeon General’s advisory on building a thriving health workforce. </p> </itemContent> </newsItem> <newsItem> <itemMeta> <itemRole>teaser</itemRole> <itemClass>text</itemClass> <title/> <deck/> </itemMeta> <itemContent> </itemContent> </newsItem> </itemSet></root>
Disallow All Ads
Content Gating
No Gating (article Unlocked/Free)
Alternative CME
Disqus Comments
Default
Use ProPublica
Hide sidebar & use full width
render the right sidebar.
Conference Recap Checkbox
Not Conference Recap
Clinical Edge
Display the Slideshow in this Article
Medscape Article
Display survey writer
Reuters content
Disable Inline Native ads
WebMD Article

How to make visits run more smoothly and be more productive

Article Type
Changed
Wed, 05/18/2022 - 11:41

We all feel pressure from not having enough time to care for our patients the way we want to.

Below I describe related challenges I frequently face during office visits. I also share strategies for making these visits go more smoothly and be more productive.

Paauw_Doug2_web.jpg
Dr. Paauw

Organ recital

Some of our patients need to share an update on all their symptoms each visit, old and new, including those that are minor or possibly concerning. I have learned over the years that, for many patients, this allows them to release the worry about symptoms.

Some symptoms are so distressing and severe that symptomatic treatment is needed, but most aren’t.

I am very honest with patients when I have no idea what is causing their symptoms. I tell them, we will watch for other clues to see if the symptom needs a workup.

One thing I don’t do, and I strongly recommend against, is doing a review of systems. This leads a patient to believe you are concerned about exploring each possible symptom, ones that they didn’t even bring up! The yield is very low, and the time commitment is great.
 

The angry patient

Imagine a scenario when you are running 15 minutes behind and, when you step into the room, your patient is angry. You are already behind, and helping the patient navigate their anger will be part of your clinic visit.

In these situations, I always address the patient’s anger immediately. Problems with getting appointments with specialists, delays in diagnostic tests, or a broken entry to the parking garage have all been sources of my patients’ frustrations.

When we have limited time, using much of the clinic visit to process frustration leads to empty clinic visits. I listen and work to empathize with the patient, often agreeing that there are so many messed up aspects of the health care system. I do not like to use the corporate “I am sad you feel that way” response, because I feel it is not helpful. Instead, I tell them how much I want to help them today in any way I can at this visit.
 

The Internet sleuth

When our patients have new symptoms, some of them will go to the Internet to try to self-diagnose. Sometimes they make a correct diagnosis, but other times consider scary diagnoses we would not consider based on their symptoms and risk factors.

In these scenarios, I always ask the patient why they think their diagnosis is accurate. Their response to this question gives me insight into where their beliefs come from and helps me understand what information I need to provide.

McMullan said physicians can be defensive, collaborative, and informative when they interact with patients about information they have found on the Internet. In the first model, the physician is authoritative. The second involves working with the patient and obtaining and analyzing information. In the third model, the physician provides reputable internet sites to patients for obtaining information.
 

‘Oh, by the way’

Patients frequently bring up sensitive topics or complicated requests after the visit has wrapped up. Topics such as insomnia, erectile dysfunction, and anxiety are often brought up with the assumption that a quick prescription is the answer. For many years, I would add time to the appointment and try to address these issues as quickly as I could. But I invariably did a poor job at helping with these problems. Now, I offer to see the patient back soon to spend an entire visit discussing the newly brought up concern. I tell them that the problem is too important to not have my full attention and focus.

Pearls

  • Empathetically listen to descriptions of symptoms, but don’t focus on fixing them.
  • Empathize with the angry patient, and move on to taking care of their medical problems.
  • Avoid the urge to address newly raised problems at the end of the visit.

Dr. Paauw is professor of medicine in the division of general internal medicine at the University of Washington, Seattle, and serves as third-year medical student clerkship director at the University of Washington. He is a member of the editorial advisory board of Internal Medicine News. Dr. Paauw has no conflicts to disclose.

Publications
Topics
Sections

We all feel pressure from not having enough time to care for our patients the way we want to.

Below I describe related challenges I frequently face during office visits. I also share strategies for making these visits go more smoothly and be more productive.

Paauw_Doug2_web.jpg
Dr. Paauw

Organ recital

Some of our patients need to share an update on all their symptoms each visit, old and new, including those that are minor or possibly concerning. I have learned over the years that, for many patients, this allows them to release the worry about symptoms.

Some symptoms are so distressing and severe that symptomatic treatment is needed, but most aren’t.

I am very honest with patients when I have no idea what is causing their symptoms. I tell them, we will watch for other clues to see if the symptom needs a workup.

One thing I don’t do, and I strongly recommend against, is doing a review of systems. This leads a patient to believe you are concerned about exploring each possible symptom, ones that they didn’t even bring up! The yield is very low, and the time commitment is great.
 

The angry patient

Imagine a scenario when you are running 15 minutes behind and, when you step into the room, your patient is angry. You are already behind, and helping the patient navigate their anger will be part of your clinic visit.

In these situations, I always address the patient’s anger immediately. Problems with getting appointments with specialists, delays in diagnostic tests, or a broken entry to the parking garage have all been sources of my patients’ frustrations.

When we have limited time, using much of the clinic visit to process frustration leads to empty clinic visits. I listen and work to empathize with the patient, often agreeing that there are so many messed up aspects of the health care system. I do not like to use the corporate “I am sad you feel that way” response, because I feel it is not helpful. Instead, I tell them how much I want to help them today in any way I can at this visit.
 

The Internet sleuth

When our patients have new symptoms, some of them will go to the Internet to try to self-diagnose. Sometimes they make a correct diagnosis, but other times consider scary diagnoses we would not consider based on their symptoms and risk factors.

In these scenarios, I always ask the patient why they think their diagnosis is accurate. Their response to this question gives me insight into where their beliefs come from and helps me understand what information I need to provide.

McMullan said physicians can be defensive, collaborative, and informative when they interact with patients about information they have found on the Internet. In the first model, the physician is authoritative. The second involves working with the patient and obtaining and analyzing information. In the third model, the physician provides reputable internet sites to patients for obtaining information.
 

‘Oh, by the way’

Patients frequently bring up sensitive topics or complicated requests after the visit has wrapped up. Topics such as insomnia, erectile dysfunction, and anxiety are often brought up with the assumption that a quick prescription is the answer. For many years, I would add time to the appointment and try to address these issues as quickly as I could. But I invariably did a poor job at helping with these problems. Now, I offer to see the patient back soon to spend an entire visit discussing the newly brought up concern. I tell them that the problem is too important to not have my full attention and focus.

Pearls

  • Empathetically listen to descriptions of symptoms, but don’t focus on fixing them.
  • Empathize with the angry patient, and move on to taking care of their medical problems.
  • Avoid the urge to address newly raised problems at the end of the visit.

Dr. Paauw is professor of medicine in the division of general internal medicine at the University of Washington, Seattle, and serves as third-year medical student clerkship director at the University of Washington. He is a member of the editorial advisory board of Internal Medicine News. Dr. Paauw has no conflicts to disclose.

We all feel pressure from not having enough time to care for our patients the way we want to.

Below I describe related challenges I frequently face during office visits. I also share strategies for making these visits go more smoothly and be more productive.

Paauw_Doug2_web.jpg
Dr. Paauw

Organ recital

Some of our patients need to share an update on all their symptoms each visit, old and new, including those that are minor or possibly concerning. I have learned over the years that, for many patients, this allows them to release the worry about symptoms.

Some symptoms are so distressing and severe that symptomatic treatment is needed, but most aren’t.

I am very honest with patients when I have no idea what is causing their symptoms. I tell them, we will watch for other clues to see if the symptom needs a workup.

One thing I don’t do, and I strongly recommend against, is doing a review of systems. This leads a patient to believe you are concerned about exploring each possible symptom, ones that they didn’t even bring up! The yield is very low, and the time commitment is great.
 

The angry patient

Imagine a scenario when you are running 15 minutes behind and, when you step into the room, your patient is angry. You are already behind, and helping the patient navigate their anger will be part of your clinic visit.

In these situations, I always address the patient’s anger immediately. Problems with getting appointments with specialists, delays in diagnostic tests, or a broken entry to the parking garage have all been sources of my patients’ frustrations.

When we have limited time, using much of the clinic visit to process frustration leads to empty clinic visits. I listen and work to empathize with the patient, often agreeing that there are so many messed up aspects of the health care system. I do not like to use the corporate “I am sad you feel that way” response, because I feel it is not helpful. Instead, I tell them how much I want to help them today in any way I can at this visit.
 

The Internet sleuth

When our patients have new symptoms, some of them will go to the Internet to try to self-diagnose. Sometimes they make a correct diagnosis, but other times consider scary diagnoses we would not consider based on their symptoms and risk factors.

In these scenarios, I always ask the patient why they think their diagnosis is accurate. Their response to this question gives me insight into where their beliefs come from and helps me understand what information I need to provide.

McMullan said physicians can be defensive, collaborative, and informative when they interact with patients about information they have found on the Internet. In the first model, the physician is authoritative. The second involves working with the patient and obtaining and analyzing information. In the third model, the physician provides reputable internet sites to patients for obtaining information.
 

‘Oh, by the way’

Patients frequently bring up sensitive topics or complicated requests after the visit has wrapped up. Topics such as insomnia, erectile dysfunction, and anxiety are often brought up with the assumption that a quick prescription is the answer. For many years, I would add time to the appointment and try to address these issues as quickly as I could. But I invariably did a poor job at helping with these problems. Now, I offer to see the patient back soon to spend an entire visit discussing the newly brought up concern. I tell them that the problem is too important to not have my full attention and focus.

Pearls

  • Empathetically listen to descriptions of symptoms, but don’t focus on fixing them.
  • Empathize with the angry patient, and move on to taking care of their medical problems.
  • Avoid the urge to address newly raised problems at the end of the visit.

Dr. Paauw is professor of medicine in the division of general internal medicine at the University of Washington, Seattle, and serves as third-year medical student clerkship director at the University of Washington. He is a member of the editorial advisory board of Internal Medicine News. Dr. Paauw has no conflicts to disclose.

Publications
Publications
Topics
Article Type
Sections
Teambase XML
<?xml version="1.0" encoding="UTF-8"?>
<!--$RCSfile: InCopy_agile.xsl,v $ $Revision: 1.35 $-->
<!--$RCSfile: drupal.xsl,v $ $Revision: 1.7 $-->
<root generator="drupal.xsl" gversion="1.7"> <header> <fileName>157920</fileName> <TBEID>0C0427F4.SIG</TBEID> <TBUniqueIdentifier>MD_0C0427F4</TBUniqueIdentifier> <newsOrJournal>News</newsOrJournal> <publisherName>Frontline Medical Communications</publisherName> <storyname>How to make visits run more smoo</storyname> <articleType>353</articleType> <TBLocation>QC Done-All Pubs</TBLocation> <QCDate>20220516T115748</QCDate> <firstPublished>20220516T121435</firstPublished> <LastPublished>20220516T121435</LastPublished> <pubStatus qcode="stat:"/> <embargoDate/> <killDate/> <CMSDate>20220516T121434</CMSDate> <articleSource/> <facebookInfo/> <meetingNumber/> <byline/> <bylineText>DOUGLAS S. PAAUW, MD</bylineText> <bylineFull>DOUGLAS S. PAAUW, MD</bylineFull> <bylineTitleText>MDedge News</bylineTitleText> <USOrGlobal/> <wireDocType/> <newsDocType>Opinion</newsDocType> <journalDocType/> <linkLabel/> <pageRange/> <citation/> <quizID/> <indexIssueDate/> <itemClass qcode="ninat:text"/> <provider qcode="provider:imng"> <name>IMNG Medical Media</name> <rightsInfo> <copyrightHolder> <name>Frontline Medical News</name> </copyrightHolder> <copyrightNotice>Copyright (c) 2015 Frontline Medical News, a Frontline Medical Communications Inc. company. All rights reserved. This material may not be published, broadcast, copied, or otherwise reproduced or distributed without the prior written permission of Frontline Medical Communications Inc.</copyrightNotice> </rightsInfo> </provider> <abstract/> <metaDescription>Below I describe related challenges I frequently face during office visits. I also share strategies for making these visits go more smoothly and be more product</metaDescription> <articlePDF/> <teaserImage>241147</teaserImage> <teaser>Some symptoms are so distressing and severe that symptomatic treatment is needed, but most aren’t.</teaser> <title>How to make visits run more smoothly and be more productive</title> <deck/> <disclaimer/> <AuthorList/> <articleURL/> <doi/> <pubMedID/> <publishXMLStatus/> <publishXMLVersion>1</publishXMLVersion> <useEISSN>0</useEISSN> <urgency/> <pubPubdateYear/> <pubPubdateMonth/> <pubPubdateDay/> <pubVolume/> <pubNumber/> <wireChannels/> <primaryCMSID/> <CMSIDs/> <keywords/> <seeAlsos/> <publications_g> <publicationData> <publicationCode>pn</publicationCode> <pubIssueName/> <pubArticleType/> <pubTopics/> <pubCategories/> <pubSections/> </publicationData> <publicationData> <publicationCode>im</publicationCode> <pubIssueName/> <pubArticleType/> <pubTopics/> <pubCategories/> <pubSections/> </publicationData> <publicationData> <publicationCode>fp</publicationCode> <pubIssueName/> <pubArticleType/> <pubTopics/> <pubCategories/> <pubSections/> </publicationData> <publicationData> <publicationCode>ob</publicationCode> <pubIssueName/> <pubArticleType/> <pubTopics/> <pubCategories/> <pubSections/> </publicationData> <publicationData> <publicationCode>chph</publicationCode> <pubIssueName/> <pubArticleType/> <pubTopics/> <pubCategories/> <pubSections/> </publicationData> </publications_g> <publications> <term>25</term> <term canonical="true">21</term> <term>15</term> <term>23</term> <term>6</term> </publications> <sections> <term canonical="true">39786</term> <term>41022</term> <term>52</term> </sections> <topics> <term>27442</term> <term canonical="true">38029</term> <term>278</term> <term>248</term> <term>280</term> </topics> <links> <link> <itemClass qcode="ninat:picture"/> <altRep contenttype="image/jpeg">images/2400b8e1.jpg</altRep> <description role="drol:caption">Dr. Paauw</description> <description role="drol:credit"/> </link> </links> </header> <itemSet> <newsItem> <itemMeta> <itemRole>Main</itemRole> <itemClass>text</itemClass> <title>How to make visits run more smoothly and be more productive</title> <deck/> </itemMeta> <itemContent> <p>We all feel pressure from not having enough time to care for our patients the way we want to.</p> <p><span class="tag metaDescription">Below I describe related challenges I frequently face during office visits. I also share strategies for making these visits go more smoothly and be more productive.</span> <br/><br/>[[{"fid":"241147","view_mode":"medstat_image_flush_left","fields":{"format":"medstat_image_flush_left","field_file_image_alt_text[und][0][value]":"Dr. Paauw","field_file_image_credit[und][0][value]":"","field_file_image_caption[und][0][value]":"Dr. Paauw"},"type":"media","attributes":{"class":"media-element file-medstat_image_flush_left"}}]]</p> <h2>Organ recital</h2> <p>Some of our patients need to share an update on all their symptoms each visit, old and new, including those that are minor or possibly concerning. I have learned over the years that, for many patients, this allows them to release the worry about symptoms. </p> <p>Some symptoms are so distressing and severe that symptomatic treatment is needed, but most aren’t. <br/><br/>I am very honest with patients when I have no idea what is causing their symptoms. I tell them, we will watch for other clues to see if the symptom needs a workup. <br/><br/>One thing I don’t do, and I strongly recommend against, is doing a review of systems. This leads a patient to believe you are concerned about exploring each possible symptom, ones that they didn’t even bring up! The yield is very low, and the time commitment is great.<br/><br/></p> <h2>The angry patient</h2> <p>Imagine a scenario when you are running 15 minutes behind and, when you step into the room, your patient is angry. You are already behind, and helping the patient navigate their anger will be part of your clinic visit. </p> <p>In these situations, I always address the patient’s anger immediately. Problems with getting appointments with specialists, delays in diagnostic tests, or a broken entry to the parking garage have all been sources of my patients’ frustrations.<br/><br/>When we have limited time, using much of the clinic visit to process frustration leads to empty clinic visits. I listen and work to empathize with the patient, often agreeing that there are so many messed up aspects of the health care system. I do not like to use the corporate “I am sad you feel that way” response, because I feel it is not helpful. Instead, I tell them how much I want to help them today in any way I can at this visit.<br/><br/></p> <h2>The Internet sleuth</h2> <p>When our patients have new symptoms, some of them will go to the Internet to try to self-diagnose. Sometimes they make a correct diagnosis, but other times consider scary diagnoses we would not consider based on their symptoms and risk factors. </p> <p>In these scenarios, I always ask the patient why they think their diagnosis is accurate. Their response to this question gives me insight into where their beliefs come from and helps me understand what information I need to provide. <br/><br/><a href="https://pubmed.ncbi.nlm.nih.gov/16406474/">McMullan said</a> physicians can be defensive, collaborative, and informative when they interact with patients about information they have found on the Internet. In the first model, the physician is authoritative. The second involves working with the patient and obtaining and analyzing information. In the third model, the physician provides reputable internet sites to patients for obtaining information.<br/><br/></p> <h2>‘Oh, by the way’</h2> <p>Patients frequently bring up sensitive topics or complicated requests after the visit has wrapped up. Topics such as insomnia, erectile dysfunction, and anxiety are often brought up with the assumption that a quick prescription is the answer. For many years, I would add time to the appointment and try to address these issues as quickly as I could. But I invariably did a poor job at helping with these problems. Now, I offer to see the patient back soon to spend an entire visit discussing the newly brought up concern. I tell them that the problem is too important to not have my full attention and focus.</p> <h2>Pearls</h2> <ul class="body"> <li>Empathetically listen to descriptions of symptoms, but don’t focus on fixing them.</li> <li>Empathize with the angry patient, and move on to taking care of their medical problems.</li> <li>Avoid the urge to address newly raised problems at the end of the visit.</li> </ul> <p> <em>Dr. Paauw is professor of medicine in the division of general internal medicine at the University of Washington, Seattle, and serves as third-year medical student clerkship director at the University of Washington. He is a member of the editorial advisory board of Internal Medicine News. Dr. Paauw has no conflicts to disclose.<span class="end"/> </em> </p> </itemContent> </newsItem> <newsItem> <itemMeta> <itemRole>teaser</itemRole> <itemClass>text</itemClass> <title/> <deck/> </itemMeta> <itemContent> </itemContent> </newsItem> </itemSet></root>
Disallow All Ads
Content Gating
No Gating (article Unlocked/Free)
Alternative CME
Disqus Comments
Default
Use ProPublica
Hide sidebar & use full width
render the right sidebar.
Conference Recap Checkbox
Not Conference Recap
Clinical Edge
Display the Slideshow in this Article
Medscape Article
Display survey writer
Reuters content
Disable Inline Native ads
WebMD Article

What’s the most likely cause of this man’s severe headaches?

Article Type
Changed
Thu, 04/21/2022 - 13:37

A 35-year-old man comes to clinic for evaluation of new, severe headaches. He reports that these started 3 days ago. His headache is worse when he stands, and resolves when he lies down. Valsalva maneuver makes the headache much worse. The headaches are present in the occipital region. He also has noticed the onset of tinnitus. A physical exam reveals that his blood pressure is 110/70 mm Hg, his pulse is 60 beats per minute, and his temperature is 36.4° C. His standing BP is 105/60 mm Hg and standing pulse is 66 bpm. Both his neurologic exam and noncontrast head CT scan are normal.


Which of the following is the most likely diagnosis?

A) Subarachnoid hemorrhage

B) POTS (Postural orthostatic tachycardia syndrome)

C) Hypnic headache

D) Spontaneous intracranial hypotension (SIH)

E) Acoustic neuroma

The most likely cause for this patient’s headaches given his set of symptoms is spontaneous intracranial hypotension. Orthostatic headaches are common with POTS, but the absence of tachycardia with standing makes this diagnosis unlikely.

Paauw_Doug2_web.jpg
Dr. Paauw

Spontaneous intracranial hypotension has symptoms that we are all familiar with in the post–lumbar puncture patient. In patients with post-LP headache, the positional nature makes it easy to diagnose. Patients who have had a lumbar puncture have a clear reason they have a cerebrospinal fluid (CSF) leak, leading to intracranial hypotension. Those with SIH do not.
 

Related research

Schievink summarized a lot of useful information in a review of patients with spontaneous intracranial hypotension.1 The incidence is about 5/100,000, with the most common age around 40 years old. The most common symptom is orthostatic headache. The headache usually occurs within 15 minutes upon standing, and many patients have the onset of headache rapidly upon standing.

Usually the headache improves with lying down, and it is often brought on with Valsalva maneuver. Many patients report headaches that are worse in the second half of the day.

Orthostatic headache occurs in almost all patients with spontaneous intracranial hypotension, but in one series it occurred only in 77% of patients with SIH.2 The patients who did not have typical headaches are more likely to have auditory symptoms such as tinnitus and muffled hearing.3

When you suspect SIH, appropriate workup is to start with brain MR imaging with contrast. Krantz and colleagues found dural enhancement was present in 83% of cases of SIH, venous distention sign in 75%, and brain sagging in 61%.4

About 10% of patients with SIH have normal brain imaging, so if the clinical features strongly suggest the diagnosis, moving on to spinal imaging with CT myelography or spinal MR are appropriate next steps.5

The causes of SIH are meningeal diverticula (usually in the thoracic or upper lumbar regions), ventral dural tears (usually from osteophytes), and cerebrospinal fluid–venous fistulas. Treatment of SIH has traditionally included a conservative approach of bed rest, oral hydration, and caffeine. The effectiveness of this is unknown, and, in one small series, 61% had headache symptoms at 6 months.6

Epidural blood patches are likely more rapidly effective than conservative therapy. In one study comparing the two treatments, Chung and colleagues found that 77% of the patients who received an epidural blood patch had complete headache relief at 4 weeks, compared with 40% of those who received conservative measures (P < .05).7
 

Clinical pearls

  • Strongly consider SIH in patients with positional headache.
  • Brain MR should be the first diagnostic test.

Dr. Paauw is professor of medicine in the division of general internal medicine at the University of Washington, Seattle, and serves as 3rd-year medical student clerkship director at the University of Washington. He is a member of the editorial advisory board of Internal Medicine News. Dr. Paauw has no conflicts to disclose. Contact him at imnews@mdedge.com.

References

1. Schievink WI. Spontaneous spinal cerebrospinal fluid leaks and intracranial hypotension. JAMA. 2006;295:2286-96.

2. Mea E et al. Headache attributed to spontaneous intracranial hypotension. Neurol Sci. 2008;29:164-65.

3. Krantz PG et al. Spontaneous Intracranial Hypotension: 10 Myths and Misperceptions. Headache. 2018;58:948-59.

4. Krantz PG et. al. Imaging signs in spontaneous intracranial hypotension: prevalence and relationship to CSF pressure. AJNR Am J Neuroradiol. 2016;37:1374-8.

5. Krantz PG et al. Spontaneous intracranial hypotension: Pathogenesis, diagnosis, and treatment. Neuroimaging Clin N Am. 2019;29:581-94.

6. Kong D-S et. al. Clinical features and long-term results of spontaneous intracranial hypotension. Neurosurgery. 2005;57:91-6.

7. Chung SJ et al. Short- and long-term outcomes of spontaneous CSF hypovolemia. Eur Neurol. 2005;54:63-7.

Publications
Topics
Sections

A 35-year-old man comes to clinic for evaluation of new, severe headaches. He reports that these started 3 days ago. His headache is worse when he stands, and resolves when he lies down. Valsalva maneuver makes the headache much worse. The headaches are present in the occipital region. He also has noticed the onset of tinnitus. A physical exam reveals that his blood pressure is 110/70 mm Hg, his pulse is 60 beats per minute, and his temperature is 36.4° C. His standing BP is 105/60 mm Hg and standing pulse is 66 bpm. Both his neurologic exam and noncontrast head CT scan are normal.


Which of the following is the most likely diagnosis?

A) Subarachnoid hemorrhage

B) POTS (Postural orthostatic tachycardia syndrome)

C) Hypnic headache

D) Spontaneous intracranial hypotension (SIH)

E) Acoustic neuroma

The most likely cause for this patient’s headaches given his set of symptoms is spontaneous intracranial hypotension. Orthostatic headaches are common with POTS, but the absence of tachycardia with standing makes this diagnosis unlikely.

Paauw_Doug2_web.jpg
Dr. Paauw

Spontaneous intracranial hypotension has symptoms that we are all familiar with in the post–lumbar puncture patient. In patients with post-LP headache, the positional nature makes it easy to diagnose. Patients who have had a lumbar puncture have a clear reason they have a cerebrospinal fluid (CSF) leak, leading to intracranial hypotension. Those with SIH do not.
 

Related research

Schievink summarized a lot of useful information in a review of patients with spontaneous intracranial hypotension.1 The incidence is about 5/100,000, with the most common age around 40 years old. The most common symptom is orthostatic headache. The headache usually occurs within 15 minutes upon standing, and many patients have the onset of headache rapidly upon standing.

Usually the headache improves with lying down, and it is often brought on with Valsalva maneuver. Many patients report headaches that are worse in the second half of the day.

Orthostatic headache occurs in almost all patients with spontaneous intracranial hypotension, but in one series it occurred only in 77% of patients with SIH.2 The patients who did not have typical headaches are more likely to have auditory symptoms such as tinnitus and muffled hearing.3

When you suspect SIH, appropriate workup is to start with brain MR imaging with contrast. Krantz and colleagues found dural enhancement was present in 83% of cases of SIH, venous distention sign in 75%, and brain sagging in 61%.4

About 10% of patients with SIH have normal brain imaging, so if the clinical features strongly suggest the diagnosis, moving on to spinal imaging with CT myelography or spinal MR are appropriate next steps.5

The causes of SIH are meningeal diverticula (usually in the thoracic or upper lumbar regions), ventral dural tears (usually from osteophytes), and cerebrospinal fluid–venous fistulas. Treatment of SIH has traditionally included a conservative approach of bed rest, oral hydration, and caffeine. The effectiveness of this is unknown, and, in one small series, 61% had headache symptoms at 6 months.6

Epidural blood patches are likely more rapidly effective than conservative therapy. In one study comparing the two treatments, Chung and colleagues found that 77% of the patients who received an epidural blood patch had complete headache relief at 4 weeks, compared with 40% of those who received conservative measures (P < .05).7
 

Clinical pearls

  • Strongly consider SIH in patients with positional headache.
  • Brain MR should be the first diagnostic test.

Dr. Paauw is professor of medicine in the division of general internal medicine at the University of Washington, Seattle, and serves as 3rd-year medical student clerkship director at the University of Washington. He is a member of the editorial advisory board of Internal Medicine News. Dr. Paauw has no conflicts to disclose. Contact him at imnews@mdedge.com.

References

1. Schievink WI. Spontaneous spinal cerebrospinal fluid leaks and intracranial hypotension. JAMA. 2006;295:2286-96.

2. Mea E et al. Headache attributed to spontaneous intracranial hypotension. Neurol Sci. 2008;29:164-65.

3. Krantz PG et al. Spontaneous Intracranial Hypotension: 10 Myths and Misperceptions. Headache. 2018;58:948-59.

4. Krantz PG et. al. Imaging signs in spontaneous intracranial hypotension: prevalence and relationship to CSF pressure. AJNR Am J Neuroradiol. 2016;37:1374-8.

5. Krantz PG et al. Spontaneous intracranial hypotension: Pathogenesis, diagnosis, and treatment. Neuroimaging Clin N Am. 2019;29:581-94.

6. Kong D-S et. al. Clinical features and long-term results of spontaneous intracranial hypotension. Neurosurgery. 2005;57:91-6.

7. Chung SJ et al. Short- and long-term outcomes of spontaneous CSF hypovolemia. Eur Neurol. 2005;54:63-7.

A 35-year-old man comes to clinic for evaluation of new, severe headaches. He reports that these started 3 days ago. His headache is worse when he stands, and resolves when he lies down. Valsalva maneuver makes the headache much worse. The headaches are present in the occipital region. He also has noticed the onset of tinnitus. A physical exam reveals that his blood pressure is 110/70 mm Hg, his pulse is 60 beats per minute, and his temperature is 36.4° C. His standing BP is 105/60 mm Hg and standing pulse is 66 bpm. Both his neurologic exam and noncontrast head CT scan are normal.


Which of the following is the most likely diagnosis?

A) Subarachnoid hemorrhage

B) POTS (Postural orthostatic tachycardia syndrome)

C) Hypnic headache

D) Spontaneous intracranial hypotension (SIH)

E) Acoustic neuroma

The most likely cause for this patient’s headaches given his set of symptoms is spontaneous intracranial hypotension. Orthostatic headaches are common with POTS, but the absence of tachycardia with standing makes this diagnosis unlikely.

Paauw_Doug2_web.jpg
Dr. Paauw

Spontaneous intracranial hypotension has symptoms that we are all familiar with in the post–lumbar puncture patient. In patients with post-LP headache, the positional nature makes it easy to diagnose. Patients who have had a lumbar puncture have a clear reason they have a cerebrospinal fluid (CSF) leak, leading to intracranial hypotension. Those with SIH do not.
 

Related research

Schievink summarized a lot of useful information in a review of patients with spontaneous intracranial hypotension.1 The incidence is about 5/100,000, with the most common age around 40 years old. The most common symptom is orthostatic headache. The headache usually occurs within 15 minutes upon standing, and many patients have the onset of headache rapidly upon standing.

Usually the headache improves with lying down, and it is often brought on with Valsalva maneuver. Many patients report headaches that are worse in the second half of the day.

Orthostatic headache occurs in almost all patients with spontaneous intracranial hypotension, but in one series it occurred only in 77% of patients with SIH.2 The patients who did not have typical headaches are more likely to have auditory symptoms such as tinnitus and muffled hearing.3

When you suspect SIH, appropriate workup is to start with brain MR imaging with contrast. Krantz and colleagues found dural enhancement was present in 83% of cases of SIH, venous distention sign in 75%, and brain sagging in 61%.4

About 10% of patients with SIH have normal brain imaging, so if the clinical features strongly suggest the diagnosis, moving on to spinal imaging with CT myelography or spinal MR are appropriate next steps.5

The causes of SIH are meningeal diverticula (usually in the thoracic or upper lumbar regions), ventral dural tears (usually from osteophytes), and cerebrospinal fluid–venous fistulas. Treatment of SIH has traditionally included a conservative approach of bed rest, oral hydration, and caffeine. The effectiveness of this is unknown, and, in one small series, 61% had headache symptoms at 6 months.6

Epidural blood patches are likely more rapidly effective than conservative therapy. In one study comparing the two treatments, Chung and colleagues found that 77% of the patients who received an epidural blood patch had complete headache relief at 4 weeks, compared with 40% of those who received conservative measures (P < .05).7
 

Clinical pearls

  • Strongly consider SIH in patients with positional headache.
  • Brain MR should be the first diagnostic test.

Dr. Paauw is professor of medicine in the division of general internal medicine at the University of Washington, Seattle, and serves as 3rd-year medical student clerkship director at the University of Washington. He is a member of the editorial advisory board of Internal Medicine News. Dr. Paauw has no conflicts to disclose. Contact him at imnews@mdedge.com.

References

1. Schievink WI. Spontaneous spinal cerebrospinal fluid leaks and intracranial hypotension. JAMA. 2006;295:2286-96.

2. Mea E et al. Headache attributed to spontaneous intracranial hypotension. Neurol Sci. 2008;29:164-65.

3. Krantz PG et al. Spontaneous Intracranial Hypotension: 10 Myths and Misperceptions. Headache. 2018;58:948-59.

4. Krantz PG et. al. Imaging signs in spontaneous intracranial hypotension: prevalence and relationship to CSF pressure. AJNR Am J Neuroradiol. 2016;37:1374-8.

5. Krantz PG et al. Spontaneous intracranial hypotension: Pathogenesis, diagnosis, and treatment. Neuroimaging Clin N Am. 2019;29:581-94.

6. Kong D-S et. al. Clinical features and long-term results of spontaneous intracranial hypotension. Neurosurgery. 2005;57:91-6.

7. Chung SJ et al. Short- and long-term outcomes of spontaneous CSF hypovolemia. Eur Neurol. 2005;54:63-7.

Publications
Publications
Topics
Article Type
Sections
Teambase XML
<?xml version="1.0" encoding="UTF-8"?>
<!--$RCSfile: InCopy_agile.xsl,v $ $Revision: 1.35 $-->
<!--$RCSfile: drupal.xsl,v $ $Revision: 1.7 $-->
<root generator="drupal.xsl" gversion="1.7"> <header> <fileName>157467</fileName> <TBEID>0C041E41.SIG</TBEID> <TBUniqueIdentifier>MD_0C041E41</TBUniqueIdentifier> <newsOrJournal>News</newsOrJournal> <publisherName>Frontline Medical Communications</publisherName> <storyname>What's the mostly cause of this</storyname> <articleType>2</articleType> <TBLocation>QC Done-All Pubs</TBLocation> <QCDate>20220419T141306</QCDate> <firstPublished>20220419T155021</firstPublished> <LastPublished>20220419T155021</LastPublished> <pubStatus qcode="stat:"/> <embargoDate/> <killDate/> <CMSDate>20220419T155021</CMSDate> <articleSource/> <facebookInfo/> <meetingNumber/> <byline/> <bylineText>DOUGLAS S. PAAUW, MD</bylineText> <bylineFull>DOUGLAS S. PAAUW, MD</bylineFull> <bylineTitleText>MDedge News</bylineTitleText> <USOrGlobal/> <wireDocType/> <newsDocType>Opinion</newsDocType> <journalDocType/> <linkLabel/> <pageRange/> <citation/> <quizID/> <indexIssueDate/> <itemClass qcode="ninat:text"/> <provider qcode="provider:imng"> <name>IMNG Medical Media</name> <rightsInfo> <copyrightHolder> <name>Frontline Medical News</name> </copyrightHolder> <copyrightNotice>Copyright (c) 2015 Frontline Medical News, a Frontline Medical Communications Inc. company. All rights reserved. This material may not be published, broadcast, copied, or otherwise reproduced or distributed without the prior written permission of Frontline Medical Communications Inc.</copyrightNotice> </rightsInfo> </provider> <abstract/> <metaDescription>A 35-year-old man comes to clinic for evaluation of new, severe headaches.</metaDescription> <articlePDF/> <teaserImage>241147</teaserImage> <teaser>A 35-year-old man is suddenly experiencing severe headaches, which get worse with standing and resolve after lying down.</teaser> <title>What’s the most likely cause of this man’s severe headaches?</title> <deck/> <disclaimer/> <AuthorList/> <articleURL/> <doi/> <pubMedID/> <publishXMLStatus/> <publishXMLVersion>1</publishXMLVersion> <useEISSN>0</useEISSN> <urgency/> <pubPubdateYear/> <pubPubdateMonth/> <pubPubdateDay/> <pubVolume/> <pubNumber/> <wireChannels/> <primaryCMSID/> <CMSIDs/> <keywords/> <seeAlsos/> <publications_g> <publicationData> <publicationCode>card</publicationCode> <pubIssueName/> <pubArticleType/> <pubTopics/> <pubCategories/> <pubSections/> </publicationData> <publicationData> <publicationCode>im</publicationCode> <pubIssueName/> <pubArticleType/> <pubTopics/> <pubCategories/> <pubSections/> </publicationData> <publicationData> <publicationCode>fp</publicationCode> <pubIssueName/> <pubArticleType/> <pubTopics/> <pubCategories/> <pubSections/> </publicationData> <publicationData> <publicationCode>nr</publicationCode> <pubIssueName/> <pubArticleType/> <pubTopics/> <pubCategories/> <pubSections/> <journalTitle>Neurology Reviews</journalTitle> <journalFullTitle>Neurology Reviews</journalFullTitle> <copyrightStatement>2018 Frontline Medical Communications Inc.,</copyrightStatement> </publicationData> </publications_g> <publications> <term>5</term> <term canonical="true">21</term> <term>15</term> <term>22</term> </publications> <sections> <term>52</term> <term canonical="true">39786</term> <term>41022</term> </sections> <topics> <term>274</term> <term>222</term> <term canonical="true">258</term> <term>194</term> <term>268</term> </topics> <links> <link> <itemClass qcode="ninat:picture"/> <altRep contenttype="image/jpeg">images/2400b8e1.jpg</altRep> <description role="drol:caption">Dr. Paauw</description> <description role="drol:credit"/> </link> </links> </header> <itemSet> <newsItem> <itemMeta> <itemRole>Main</itemRole> <itemClass>text</itemClass> <title>What’s the most likely cause of this man’s severe headaches?</title> <deck/> </itemMeta> <itemContent> <p><span class="tag metaDescription">A 35-year-old man comes to clinic for evaluation of new, severe headaches.</span> He reports that these started 3 days ago. His headache is worse when he stands, and resolves when he lies down. Valsalva maneuver makes the headache much worse. The headaches are present in the occipital region. He has also has noticed the onset of tinnitus. A physical exam reveals that his blood pressure is 110/70 mm Hg, his pulse is 60 beats per minute, and his temperature is 36.4° C. His standing BP is 105/60 mm Hg and standing pulse is 66 bpm. Both his neurologic exam and noncontrast head CT scan are normal.<br/><br/><br/><br/>Which of the following is the most likely diagnosis?<br/><br/>A) Subarachnoid hemorrhage<br/><br/>B) POTS (Postural orthostatic tachycardia syndrome)<br/><br/>C) Hypnic headache<br/><br/>D) Spontaneous intracranial hypotension (SIH)<br/><br/>E) Acoustic neuroma</p> <p>The most likely cause for this patient’s headaches given his set of symptoms is spontaneous intracranial hypotension. Orthostatic headaches are common with POTS, but the absence of tachycardia with standing makes this diagnosis unlikely. </p> <p>[[{"fid":"241147","view_mode":"medstat_image_flush_left","fields":{"format":"medstat_image_flush_left","field_file_image_alt_text[und][0][value]":"Dr. Paauw","field_file_image_credit[und][0][value]":"","field_file_image_caption[und][0][value]":"Dr. Paauw"},"type":"media","attributes":{"class":"media-element file-medstat_image_flush_left"}}]]Spontaneous intracranial hypotension has symptoms that we are all familiar with in the post–lumbar puncture patient. In patients with post-LP headache, the positional nature makes it easy to diagnose. Patients who have had a lumbar puncture have a clear reason they have a cerebrospinal fluid (CSK) leak, leading to intracranial hypotension. Those with SIH do not.<br/><br/></p> <h2>Related research</h2> <p>Schievink summarized a lot of useful information in a review of patients with spontaneous intracranial hypotension.<sup>1</sup> The incidence is about 5/100,000, with the most common age around 40 years old. The most common symptom is orthostatic headache. The headache usually occurs within 15 minutes upon standing, and many patients have the onset of headache rapidly upon standing. </p> <p>Usually the headache improves with lying down, and it is often brought on with Valsalva maneuver. Many patients report headaches that are worse in the second half of the day.<br/><br/>Orthostatic headache occurs in almost all patients with spontaneous intracranial hypotension, but in one series it occurred only in 77% of patients with SIH.<sup>2</sup> The patients who did not have typical headaches are more likely to have auditory symptoms such as tinnitus and muffled hearing.<sup>3</sup> <br/><br/>When you suspect SIH, appropriate workup is to start with brain MR imaging with contrast. Krantz and colleagues found dural enhancement was present in 83% of cases of SIH, venous distention sign in 75%, and brain sagging in 61%.<sup>4</sup> <br/><br/>About 10% of patients with SIH have normal brain imaging, so if the clinical features strongly suggest the diagnosis, moving on to spinal imaging with CT myelography or spinal MR are appropriate next steps.<sup>5</sup> <br/><br/>The causes of SIH are meningeal diverticula (usually in the thoracic or upper lumbar regions), ventral dural tears (usually from osteophytes), and cerebrospinal fluid–venous fistulas. Treatment of SIH has traditionally included a conservative approach of bed rest, oral hydration, and caffeine. The effectiveness of this is unknown, and, in one small series, 61% had headache symptoms at 6 months.<sup>6</sup> <br/><br/>Epidural blood patches are likely more rapidly effective than conservative therapy. In one study comparing the two treatments, Chung and colleagues found that 77% of the patients who received an epidural blood patch had complete headache relief at 4 weeks, compared with 40% of those who received conservative measures (<em>P</em> &lt; .05).<sup>7<br/><br/></sup></p> <h2>Clinical pearls</h2> <ul class="body"> <li>Strongly consider SIH in patients with positional headache.</li> <li>Brain MR should be the first diagnostic test.</li> </ul> <p> <em>Dr. Paauw is professor of medicine in the division of general internal medicine at the University of Washington, Seattle, and serves as 3rd-year medical student clerkship director at the University of Washington. He is a member of the editorial advisory board of Internal Medicine News. Dr. Paauw has no conflicts to disclose. Contact him at <a href="mailto:imnews%40mdedge.com?subject=">imnews@mdedge.com</a>.</em> </p> <h2>References</h2> <p>1. Schievink WI. Spontaneous spinal cerebrospinal fluid leaks and intracranial hypotension. <a href="https://pubmed.ncbi.nlm.nih.gov/16705110/">JAMA. 2006;295:2286-96</a>.<br/><br/>2. Mea E et al. Headache attributed to spontaneous intracranial hypotension. <a href="https://pubmed.ncbi.nlm.nih.gov/18545924/">Neurol Sci. 2008;29:164-65</a>.<br/><br/>3. Krantz PG et al. Spontaneous Intracranial Hypotension: 10 Myths and Misperceptions. <a href="https://pubmed.ncbi.nlm.nih.gov/29797515/">Headache. 2018;58:948-59</a>.<br/><br/>4. Krantz PG et. al. Imaging signs in spontaneous intracranial hypotension: prevalence and relationship to CSF pressure. <a href="https://pubmed.ncbi.nlm.nih.gov/26869465/">AJNR Am J Neuroradiol. 2016;37:1374-8</a>.<br/><br/>5. Krantz PG et al. Spontaneous intracranial hypotension: Pathogenesis, diagnosis, and treatment. <a href="https://pubmed.ncbi.nlm.nih.gov/31677732/">Neuroimaging Clin N Am. 2019;29:581-94</a>.<br/><br/>6. Kong D-S et. al. Clinical features and long-term results of spontaneous intracranial hypotension. <a href="https://pubmed.ncbi.nlm.nih.gov/15987544/">Neurosurgery. 2005;57:91-6</a>.<br/><br/>7. Chung SJ et al. Short- and long-term outcomes of spontaneous CSF hypovolemia. <a href="https://pubmed.ncbi.nlm.nih.gov/16118499/">Eur Neurol. 2005;54:63-7</a>.</p> </itemContent> </newsItem> <newsItem> <itemMeta> <itemRole>teaser</itemRole> <itemClass>text</itemClass> <title/> <deck/> </itemMeta> <itemContent> </itemContent> </newsItem> </itemSet></root>
Disallow All Ads
Content Gating
No Gating (article Unlocked/Free)
Alternative CME
Disqus Comments
Default
Use ProPublica
Hide sidebar & use full width
render the right sidebar.
Conference Recap Checkbox
Not Conference Recap
Clinical Edge
Display the Slideshow in this Article
Medscape Article
Display survey writer
Reuters content
Disable Inline Native ads
WebMD Article

Practice guidelines highlights from past year

Article Type
Changed
Fri, 03/04/2022 - 10:47

A 64-year-old woman is evaluated for fever, diarrhea, and abdominal pain. She was diagnosed with Clostridioides difficile 2 months ago and completed a 10-day course of vancomycin. Her stool toxin test is positive for Clostridioides difficile (C. diff). Based on the most recent Infectious Diseases Society of America (IDSA) guidelines, what would be the preferred therapy?

A) Metronidazole

B) Fidaxomicin + bezlotoxumab

C) Vancomycin

D) Fecal microbiota transplant

The recommendations from the 2021 guidelines would be to treat with fidaxomicin and add bezlotoxumab.1 The guidelines highlight the following changes:

  • In patients with an initial Clostridioides difficile infections (CDI) fidaxomicin is preferred over vancomycin.
  • In patients with a recurrent CDI episode, fidaxomicin is favored over vancomycin. For patients with multiple recurrences, vancomycin in a tapered and pulsed regimen, vancomycin followed by rifaximin, and fecal microbiota transplantation are options in addition to fidaxomicin.
  • Addition of bezlotoxumab to standard of care antibiotics is recommended for recurrence of CDI within the first 6 months over standard of care antibiotics alone

Paauw_Doug_SEATTLE_2019_web2.jpg
Dr. Douglas S. Paauw

The feasibility of these recommendations is up for debate. The cost of a course of fidaxomicin is $2,800, and the cost of bezlotoxumab is about $4,500. Cost effectiveness studies that helped drive the recommendations show a savings by reducing future hospitalizations for C. diff.2 Unfortunately, this enthusiasm is not shared by many insurance companies for outpatient treatment.

Knee osteoarthritis

I will save you the excitement of the new acromegaly guidelines and focus on something we see all the time: knee osteoarthritis. The American Academy of Orthopedic Surgeons has released guidelines for this condition.3 The useful points I found were as follows:

  • Topical application of nonsteroidal anti-inflammatory drugs (e.g., diclofenac) should be used to improve function and quality of life in patients with knee osteoarthritis.
  • Exercise routines (i.e, supervised, unsupervised, and/or aquatic) are recommended versus no exercise for improving pain and function in patients with knee osteoarthritis.
  • Not recommended is the use of oral narcotics (including tramadol), as they are not effective at improving pain or function, and their use results in a significant increased risk of adverse events.
  • Not recommended for routine use in symptomatic knee osteoarthritis is intra-articular injection of hyaluronic acid.

I was happy to see topical NSAIDS recommended, as they are a much safer option in older patients than oral NSAIDS (which were also recommended). The recommendation against narcotics, including tramadol, is a shift from the recommendation of tramadol in the 2013 guidelines.4 Acetaminophen was enthusiastically recommended, and is still worth a try.

Sexually transmitted infections

The CDC published new guidelines on sexually transmitted infections last year, and there were several changes from previous guidelines that were worth highlighting.These include the following:
  • The dosing for the treatment of gonorrhea has increased to 500 mg of ceftriaxone (was 250 mg in 2015 guidelines), with a dose of 1 gram for patients who weigh more than 150 kg.
  • Chlamydia infections should be treated with a 7-day course of doxycycline as the preferred antibiotic, except in pregnant women (where azithromycin is recommended).
  • Herpes simplex virus 2 recurrences can be treated with twice-daily dosing of 800 mg of acyclovir for 5 days, or acyclovir 800 mg three times a day for 2 days. The shortest course for recurrence is famciclovir 1 gram twice a day for 1 day.
  • The Centers for Disease Control and Prevention has removed the recommendation for avoidance of alcohol when taking metronidazole.

I hope these highlights of guidelines for common issues we see are helpful!

Dr. Paauw is professor of medicine in the division of general internal medicine at the University of Washington, Seattle, and serves as third-year medical student clerkship director at the University of Washington. He is a member of the editorial advisory board of Internal Medicine News. Dr. Paauw has no conflicts to disclose. Contact him at imnews@mdedge.com.

References

1. Johnson S et al. Clinical practice guideline by the Infectious Diseases Society of America (IDSA) and Society for Healthcare Epidemiology of America (SHEA): 2021 Focused update guidelines on management of Clostridioides difficile Infection in adults. Clin Infect Dis. 2021 Sep 7;73(5):e1029-e1044.

2. Pabhu VS et al. Cost-effectiveness of bezlotoxumab compared with placebo for the prevention of recurrent Clostridium difficile infection. Clin Infect Dis. 2018 Feb 1;66(3):355-62.

3. American Academy of Orthopaedic Surgeons: Management of osteoarthritis of the knee (non-arthroplasty) – Evidence-based clinical practice guideline (2021 Aug 31. https://www.aaos.org/oak3cpg).

4. Jevsevar DS. Treatment of osteoarthritis of the knee: Evidence-based guideline, 2nd edition. J Am Acad Orthop Surg. 2013: Sep;21(9):571-6.

5. Sexually transmitted infections treatment guidelines, 2021 recommendations and reports. MMWR 2021 Jul 23;70(4):1-187.

Publications
Topics
Sections

A 64-year-old woman is evaluated for fever, diarrhea, and abdominal pain. She was diagnosed with Clostridioides difficile 2 months ago and completed a 10-day course of vancomycin. Her stool toxin test is positive for Clostridioides difficile (C. diff). Based on the most recent Infectious Diseases Society of America (IDSA) guidelines, what would be the preferred therapy?

A) Metronidazole

B) Fidaxomicin + bezlotoxumab

C) Vancomycin

D) Fecal microbiota transplant

The recommendations from the 2021 guidelines would be to treat with fidaxomicin and add bezlotoxumab.1 The guidelines highlight the following changes:

  • In patients with an initial Clostridioides difficile infections (CDI) fidaxomicin is preferred over vancomycin.
  • In patients with a recurrent CDI episode, fidaxomicin is favored over vancomycin. For patients with multiple recurrences, vancomycin in a tapered and pulsed regimen, vancomycin followed by rifaximin, and fecal microbiota transplantation are options in addition to fidaxomicin.
  • Addition of bezlotoxumab to standard of care antibiotics is recommended for recurrence of CDI within the first 6 months over standard of care antibiotics alone

Paauw_Doug_SEATTLE_2019_web2.jpg
Dr. Douglas S. Paauw

The feasibility of these recommendations is up for debate. The cost of a course of fidaxomicin is $2,800, and the cost of bezlotoxumab is about $4,500. Cost effectiveness studies that helped drive the recommendations show a savings by reducing future hospitalizations for C. diff.2 Unfortunately, this enthusiasm is not shared by many insurance companies for outpatient treatment.

Knee osteoarthritis

I will save you the excitement of the new acromegaly guidelines and focus on something we see all the time: knee osteoarthritis. The American Academy of Orthopedic Surgeons has released guidelines for this condition.3 The useful points I found were as follows:

  • Topical application of nonsteroidal anti-inflammatory drugs (e.g., diclofenac) should be used to improve function and quality of life in patients with knee osteoarthritis.
  • Exercise routines (i.e, supervised, unsupervised, and/or aquatic) are recommended versus no exercise for improving pain and function in patients with knee osteoarthritis.
  • Not recommended is the use of oral narcotics (including tramadol), as they are not effective at improving pain or function, and their use results in a significant increased risk of adverse events.
  • Not recommended for routine use in symptomatic knee osteoarthritis is intra-articular injection of hyaluronic acid.

I was happy to see topical NSAIDS recommended, as they are a much safer option in older patients than oral NSAIDS (which were also recommended). The recommendation against narcotics, including tramadol, is a shift from the recommendation of tramadol in the 2013 guidelines.4 Acetaminophen was enthusiastically recommended, and is still worth a try.

Sexually transmitted infections

The CDC published new guidelines on sexually transmitted infections last year, and there were several changes from previous guidelines that were worth highlighting.These include the following:
  • The dosing for the treatment of gonorrhea has increased to 500 mg of ceftriaxone (was 250 mg in 2015 guidelines), with a dose of 1 gram for patients who weigh more than 150 kg.
  • Chlamydia infections should be treated with a 7-day course of doxycycline as the preferred antibiotic, except in pregnant women (where azithromycin is recommended).
  • Herpes simplex virus 2 recurrences can be treated with twice-daily dosing of 800 mg of acyclovir for 5 days, or acyclovir 800 mg three times a day for 2 days. The shortest course for recurrence is famciclovir 1 gram twice a day for 1 day.
  • The Centers for Disease Control and Prevention has removed the recommendation for avoidance of alcohol when taking metronidazole.

I hope these highlights of guidelines for common issues we see are helpful!

Dr. Paauw is professor of medicine in the division of general internal medicine at the University of Washington, Seattle, and serves as third-year medical student clerkship director at the University of Washington. He is a member of the editorial advisory board of Internal Medicine News. Dr. Paauw has no conflicts to disclose. Contact him at imnews@mdedge.com.

References

1. Johnson S et al. Clinical practice guideline by the Infectious Diseases Society of America (IDSA) and Society for Healthcare Epidemiology of America (SHEA): 2021 Focused update guidelines on management of Clostridioides difficile Infection in adults. Clin Infect Dis. 2021 Sep 7;73(5):e1029-e1044.

2. Pabhu VS et al. Cost-effectiveness of bezlotoxumab compared with placebo for the prevention of recurrent Clostridium difficile infection. Clin Infect Dis. 2018 Feb 1;66(3):355-62.

3. American Academy of Orthopaedic Surgeons: Management of osteoarthritis of the knee (non-arthroplasty) – Evidence-based clinical practice guideline (2021 Aug 31. https://www.aaos.org/oak3cpg).

4. Jevsevar DS. Treatment of osteoarthritis of the knee: Evidence-based guideline, 2nd edition. J Am Acad Orthop Surg. 2013: Sep;21(9):571-6.

5. Sexually transmitted infections treatment guidelines, 2021 recommendations and reports. MMWR 2021 Jul 23;70(4):1-187.

A 64-year-old woman is evaluated for fever, diarrhea, and abdominal pain. She was diagnosed with Clostridioides difficile 2 months ago and completed a 10-day course of vancomycin. Her stool toxin test is positive for Clostridioides difficile (C. diff). Based on the most recent Infectious Diseases Society of America (IDSA) guidelines, what would be the preferred therapy?

A) Metronidazole

B) Fidaxomicin + bezlotoxumab

C) Vancomycin

D) Fecal microbiota transplant

The recommendations from the 2021 guidelines would be to treat with fidaxomicin and add bezlotoxumab.1 The guidelines highlight the following changes:

  • In patients with an initial Clostridioides difficile infections (CDI) fidaxomicin is preferred over vancomycin.
  • In patients with a recurrent CDI episode, fidaxomicin is favored over vancomycin. For patients with multiple recurrences, vancomycin in a tapered and pulsed regimen, vancomycin followed by rifaximin, and fecal microbiota transplantation are options in addition to fidaxomicin.
  • Addition of bezlotoxumab to standard of care antibiotics is recommended for recurrence of CDI within the first 6 months over standard of care antibiotics alone

Paauw_Doug_SEATTLE_2019_web2.jpg
Dr. Douglas S. Paauw

The feasibility of these recommendations is up for debate. The cost of a course of fidaxomicin is $2,800, and the cost of bezlotoxumab is about $4,500. Cost effectiveness studies that helped drive the recommendations show a savings by reducing future hospitalizations for C. diff.2 Unfortunately, this enthusiasm is not shared by many insurance companies for outpatient treatment.

Knee osteoarthritis

I will save you the excitement of the new acromegaly guidelines and focus on something we see all the time: knee osteoarthritis. The American Academy of Orthopedic Surgeons has released guidelines for this condition.3 The useful points I found were as follows:

  • Topical application of nonsteroidal anti-inflammatory drugs (e.g., diclofenac) should be used to improve function and quality of life in patients with knee osteoarthritis.
  • Exercise routines (i.e, supervised, unsupervised, and/or aquatic) are recommended versus no exercise for improving pain and function in patients with knee osteoarthritis.
  • Not recommended is the use of oral narcotics (including tramadol), as they are not effective at improving pain or function, and their use results in a significant increased risk of adverse events.
  • Not recommended for routine use in symptomatic knee osteoarthritis is intra-articular injection of hyaluronic acid.

I was happy to see topical NSAIDS recommended, as they are a much safer option in older patients than oral NSAIDS (which were also recommended). The recommendation against narcotics, including tramadol, is a shift from the recommendation of tramadol in the 2013 guidelines.4 Acetaminophen was enthusiastically recommended, and is still worth a try.

Sexually transmitted infections

The CDC published new guidelines on sexually transmitted infections last year, and there were several changes from previous guidelines that were worth highlighting.These include the following:
  • The dosing for the treatment of gonorrhea has increased to 500 mg of ceftriaxone (was 250 mg in 2015 guidelines), with a dose of 1 gram for patients who weigh more than 150 kg.
  • Chlamydia infections should be treated with a 7-day course of doxycycline as the preferred antibiotic, except in pregnant women (where azithromycin is recommended).
  • Herpes simplex virus 2 recurrences can be treated with twice-daily dosing of 800 mg of acyclovir for 5 days, or acyclovir 800 mg three times a day for 2 days. The shortest course for recurrence is famciclovir 1 gram twice a day for 1 day.
  • The Centers for Disease Control and Prevention has removed the recommendation for avoidance of alcohol when taking metronidazole.

I hope these highlights of guidelines for common issues we see are helpful!

Dr. Paauw is professor of medicine in the division of general internal medicine at the University of Washington, Seattle, and serves as third-year medical student clerkship director at the University of Washington. He is a member of the editorial advisory board of Internal Medicine News. Dr. Paauw has no conflicts to disclose. Contact him at imnews@mdedge.com.

References

1. Johnson S et al. Clinical practice guideline by the Infectious Diseases Society of America (IDSA) and Society for Healthcare Epidemiology of America (SHEA): 2021 Focused update guidelines on management of Clostridioides difficile Infection in adults. Clin Infect Dis. 2021 Sep 7;73(5):e1029-e1044.

2. Pabhu VS et al. Cost-effectiveness of bezlotoxumab compared with placebo for the prevention of recurrent Clostridium difficile infection. Clin Infect Dis. 2018 Feb 1;66(3):355-62.

3. American Academy of Orthopaedic Surgeons: Management of osteoarthritis of the knee (non-arthroplasty) – Evidence-based clinical practice guideline (2021 Aug 31. https://www.aaos.org/oak3cpg).

4. Jevsevar DS. Treatment of osteoarthritis of the knee: Evidence-based guideline, 2nd edition. J Am Acad Orthop Surg. 2013: Sep;21(9):571-6.

5. Sexually transmitted infections treatment guidelines, 2021 recommendations and reports. MMWR 2021 Jul 23;70(4):1-187.

Publications
Publications
Topics
Article Type
Sections
Disallow All Ads
Content Gating
No Gating (article Unlocked/Free)
Alternative CME
Disqus Comments
Default
Use ProPublica
Hide sidebar & use full width
render the right sidebar.
Conference Recap Checkbox
Not Conference Recap
Clinical Edge
Display the Slideshow in this Article
Medscape Article
Display survey writer
Reuters content
Disable Inline Native ads
WebMD Article

Tips for connecting with your patients

Article Type
Changed
Tue, 02/15/2022 - 15:34

It is a tough time to be a doctor. With the stresses of the pandemic, the continued unfettered rise of insurance company BS, and so many medical groups being bought up that we often don’t even know who makes the decisions, the patient can sometimes be hidden in the equation.

What are ways that we can connect well with our patients so that both the patient and the physician are lifted up by the relationship?

Paauw_Doug2_web.jpg
Dr. Paauw

Be curious

When physicians are curious about why patients have symptoms, how those symptoms will affect their lives, and how worried the patient is about them, patients feel cared about.

Ascertaining how concerned patients are about their symptoms will help you make decisions on whether symptoms you are not concerned about actually need to be treated.
 

Limit use of EHRs when possible

Use of the electronic health record during visits is essential, but focusing on it too much can put a barrier between the physician and the patient.

Marmor and colleagues found there is an inverse relationship between time spent on the EHR by a patient’s physician and the patient’s satisfaction.1

Eye contact with the patient is important, especially when patients are sharing concerns they are scared about and upsetting experiences. There can be awkward pauses when looking things up on the EHR. Fill those pauses by explaining to the patient what you are doing, or chatting with the patient.
 

Consider teaching medical students

When a medical student works with you, it doubles the time the patient gets with a concerned listener. Students also can do a great job with timely follow-up and checking in with worried patients.

By having the student present in the clinic room, with the patient present, the patient can really feel heard. The student shares all the details the patient shared, and now their physician is hearing an organized, thoughtful report of the patients concerns.

In fact, I was involved in a study that showed that patients preferred in room presentations, and that they were more satisfied when students presented in the room.2
 

Use healing words

Some words carry loaded emotions. The word chronic, for example, has negative connotations, whereas the term persisting does not.

I will often ask patients how long they have been suffering from a symptom to imply my concern for what they are going through. The term “chief complaint” is outdated, and upsets patients when they see it in their medical record.

As a patient of mine once said to me: “I never complained about that problem, I just brought it to your attention.” No one wants to be seen as a complainer. Substituting the word concern for complaint works well.
 

Explain as you examine

People love to hear the term normal. When you are examining a patient, let them know when findings are normal.

I also find it helpful to explain to patients why I am doing certain physical exam maneuvers. This helps them assess how thorough we are in our thought process.

When patients feel their physicians are thorough, they have more confidence in them.
 

In summary

  • Be curious.
  • Do not overly focus on the EHR.
  • Consider teaching a medical student.
  • Be careful of word choice.
  • “Overexplain” the physical exam.

Dr. Paauw is professor of medicine in the division of general internal medicine at the University of Washington, Seattle, and serves as 3rd-year medical student clerkship director at the University of Washington. He is a member of the editorial advisory board of Internal Medicine News. Dr. Paauw has no conflicts to disclose. Contact him at imnews@mdedge.com.

References

1. Marmor RA et al. Appl Clin Inform. 2018 Jan;9(1):11-4.

2. Rogers HD et al. Acad Med. 2003 Sep;78(9):945-9.

Publications
Topics
Sections

It is a tough time to be a doctor. With the stresses of the pandemic, the continued unfettered rise of insurance company BS, and so many medical groups being bought up that we often don’t even know who makes the decisions, the patient can sometimes be hidden in the equation.

What are ways that we can connect well with our patients so that both the patient and the physician are lifted up by the relationship?

Paauw_Doug2_web.jpg
Dr. Paauw

Be curious

When physicians are curious about why patients have symptoms, how those symptoms will affect their lives, and how worried the patient is about them, patients feel cared about.

Ascertaining how concerned patients are about their symptoms will help you make decisions on whether symptoms you are not concerned about actually need to be treated.
 

Limit use of EHRs when possible

Use of the electronic health record during visits is essential, but focusing on it too much can put a barrier between the physician and the patient.

Marmor and colleagues found there is an inverse relationship between time spent on the EHR by a patient’s physician and the patient’s satisfaction.1

Eye contact with the patient is important, especially when patients are sharing concerns they are scared about and upsetting experiences. There can be awkward pauses when looking things up on the EHR. Fill those pauses by explaining to the patient what you are doing, or chatting with the patient.
 

Consider teaching medical students

When a medical student works with you, it doubles the time the patient gets with a concerned listener. Students also can do a great job with timely follow-up and checking in with worried patients.

By having the student present in the clinic room, with the patient present, the patient can really feel heard. The student shares all the details the patient shared, and now their physician is hearing an organized, thoughtful report of the patients concerns.

In fact, I was involved in a study that showed that patients preferred in room presentations, and that they were more satisfied when students presented in the room.2
 

Use healing words

Some words carry loaded emotions. The word chronic, for example, has negative connotations, whereas the term persisting does not.

I will often ask patients how long they have been suffering from a symptom to imply my concern for what they are going through. The term “chief complaint” is outdated, and upsets patients when they see it in their medical record.

As a patient of mine once said to me: “I never complained about that problem, I just brought it to your attention.” No one wants to be seen as a complainer. Substituting the word concern for complaint works well.
 

Explain as you examine

People love to hear the term normal. When you are examining a patient, let them know when findings are normal.

I also find it helpful to explain to patients why I am doing certain physical exam maneuvers. This helps them assess how thorough we are in our thought process.

When patients feel their physicians are thorough, they have more confidence in them.
 

In summary

  • Be curious.
  • Do not overly focus on the EHR.
  • Consider teaching a medical student.
  • Be careful of word choice.
  • “Overexplain” the physical exam.

Dr. Paauw is professor of medicine in the division of general internal medicine at the University of Washington, Seattle, and serves as 3rd-year medical student clerkship director at the University of Washington. He is a member of the editorial advisory board of Internal Medicine News. Dr. Paauw has no conflicts to disclose. Contact him at imnews@mdedge.com.

References

1. Marmor RA et al. Appl Clin Inform. 2018 Jan;9(1):11-4.

2. Rogers HD et al. Acad Med. 2003 Sep;78(9):945-9.

It is a tough time to be a doctor. With the stresses of the pandemic, the continued unfettered rise of insurance company BS, and so many medical groups being bought up that we often don’t even know who makes the decisions, the patient can sometimes be hidden in the equation.

What are ways that we can connect well with our patients so that both the patient and the physician are lifted up by the relationship?

Paauw_Doug2_web.jpg
Dr. Paauw

Be curious

When physicians are curious about why patients have symptoms, how those symptoms will affect their lives, and how worried the patient is about them, patients feel cared about.

Ascertaining how concerned patients are about their symptoms will help you make decisions on whether symptoms you are not concerned about actually need to be treated.
 

Limit use of EHRs when possible

Use of the electronic health record during visits is essential, but focusing on it too much can put a barrier between the physician and the patient.

Marmor and colleagues found there is an inverse relationship between time spent on the EHR by a patient’s physician and the patient’s satisfaction.1

Eye contact with the patient is important, especially when patients are sharing concerns they are scared about and upsetting experiences. There can be awkward pauses when looking things up on the EHR. Fill those pauses by explaining to the patient what you are doing, or chatting with the patient.
 

Consider teaching medical students

When a medical student works with you, it doubles the time the patient gets with a concerned listener. Students also can do a great job with timely follow-up and checking in with worried patients.

By having the student present in the clinic room, with the patient present, the patient can really feel heard. The student shares all the details the patient shared, and now their physician is hearing an organized, thoughtful report of the patients concerns.

In fact, I was involved in a study that showed that patients preferred in room presentations, and that they were more satisfied when students presented in the room.2
 

Use healing words

Some words carry loaded emotions. The word chronic, for example, has negative connotations, whereas the term persisting does not.

I will often ask patients how long they have been suffering from a symptom to imply my concern for what they are going through. The term “chief complaint” is outdated, and upsets patients when they see it in their medical record.

As a patient of mine once said to me: “I never complained about that problem, I just brought it to your attention.” No one wants to be seen as a complainer. Substituting the word concern for complaint works well.
 

Explain as you examine

People love to hear the term normal. When you are examining a patient, let them know when findings are normal.

I also find it helpful to explain to patients why I am doing certain physical exam maneuvers. This helps them assess how thorough we are in our thought process.

When patients feel their physicians are thorough, they have more confidence in them.
 

In summary

  • Be curious.
  • Do not overly focus on the EHR.
  • Consider teaching a medical student.
  • Be careful of word choice.
  • “Overexplain” the physical exam.

Dr. Paauw is professor of medicine in the division of general internal medicine at the University of Washington, Seattle, and serves as 3rd-year medical student clerkship director at the University of Washington. He is a member of the editorial advisory board of Internal Medicine News. Dr. Paauw has no conflicts to disclose. Contact him at imnews@mdedge.com.

References

1. Marmor RA et al. Appl Clin Inform. 2018 Jan;9(1):11-4.

2. Rogers HD et al. Acad Med. 2003 Sep;78(9):945-9.

Publications
Publications
Topics
Article Type
Sections
Disallow All Ads
Content Gating
No Gating (article Unlocked/Free)
Alternative CME
Disqus Comments
Default
Use ProPublica
Hide sidebar & use full width
render the right sidebar.
Conference Recap Checkbox
Not Conference Recap
Clinical Edge
Display the Slideshow in this Article
Medscape Article
Display survey writer
Reuters content
Disable Inline Native ads
WebMD Article

My favorite physical exam pearls

Article Type
Changed
Thu, 01/06/2022 - 13:59

 

I would like to start the new year off by returning to the past – when the physical exam was emphasized and utilized in decision making. I think a big reason that its use has diminished in recent years is due to the physical exam not having been emphasized in training.

For those seeking to increase their comfort with conducting the physical exam, below are several methods I have found helpful to use in practice.
 

Examining the pharynx

We were usually taught to ask the patient to say ahhh, with or without a nasty tongue depressor.

Paauw_Doug_SEATTLE_2019_web2.jpg
Dr. Douglas S. Paauw

When I was on my pediatrics rotation, I was taught to ask the patients to roar like a lion, which always gave a nice look at their posterior pharynx. The kids also really liked doing this, but it might seem a little strange to ask adults to do this.

A technique I have found that works well with adults is to ask them to yawn. I have found that this get me a great look at the pharynx for about half of my patients.
 

Auscultatory percussion for pleural effusions

Guarino and colleagues described a technique that is easily mastered and very effective for determining the presence of pleural effusions.1 It involves placing the stethoscope 3 cm below the last rib in the mid clavicular line and tapping from the apex down to the last rib.

For patients without effusion, a sharp change to a loud percussion note will occur at the last rib.

If the patient has an effusion, the loud percussion note will start at the top of the effusion.

This method was remarkably successful at finding pleural effusions. In the study, Dr. Guarino found a sensitivity of 96% and a specificity of 100%.
 

Physical exam for anemia

Look at the nails and see if they look pale. How can we do this?

The first step is to know what your own hematocrit is. You can then compare the color of your nail to that of the patient.

If you have a normal hematocrit and the patient’s nail bed color is lighter than yours, the patient likely has anemia. If you do this frequently, you will get good at estimating hematocrit. This is especially important if you do not have labs readily available.

Another way to assess for anemia is to look at the color tint of the lower conjunctiva. The best way to look for this is to look at whether there is a generous amount of visible capillaries in the lower conjunctiva. Patients without anemia have a darker red color because of these vessels, whereas patients with anemia are a lighter pink.

Strobach and colleagues2 looked at both nail bed rubor and color tint of the lower conjunctiva and found that both reliably predicted presence and degree of anemia.

 

 

Determining if clubbing is present

Most physicians are aware of Shamroth sign, and use it to evaluate for clubbing. Shamroth sign is the loss of the diamond that is created by placing the back surfaces of opposite terminal phalanges together.

I have found that it’s easier to diagnose mild clubbing by looking at the finger in profile. If the ratio of the distal phalangeal depth compared to the depth across the distal interphalangeal joint is greater than 1:1, then clubbing is present.3

Pearls

1. Have the patient try yawning to better see the pharynx without using a tongue blade.

2. Try the technique of auscultatory percussion to be more accurate at picking up pleural effusions.

3. Know your hematocrit, so you can better use color shade to assess for anemia.

4. Try looking at fingers in profile to pick up clubbing.

Dr. Paauw is professor of medicine in the division of general internal medicine at the University of Washington, Seattle, and serves as 3rd-year medical student clerkship director at the University of Washington. He is a member of the editorial advisory board of Internal Medicine News. Dr. Paauw has no conflicts to disclose. Contact him at imnews@mdedge.com.

References

1. Guarino JR and Guarino JC. Auscultatory percussion: A simple method to detect pleural effusion. J Gen Intern Med. 1994 Feb;9(2):71-4.

2. Strobach RS et al. The value of the physical examination in the diagnosis of anemia. Correlation of the physical findings and the hemoglobin concentration. Arch Intern Med. 1988 Apr;148(4):831-2.

3. Spicknall KE et al. Clubbing: an update on diagnosis, differential diagnosis, pathophysiology, and clinical relevance. J Am Acad Dermatol. 2005 Jun;52(6):1020-8.

Publications
Topics
Sections

 

I would like to start the new year off by returning to the past – when the physical exam was emphasized and utilized in decision making. I think a big reason that its use has diminished in recent years is due to the physical exam not having been emphasized in training.

For those seeking to increase their comfort with conducting the physical exam, below are several methods I have found helpful to use in practice.
 

Examining the pharynx

We were usually taught to ask the patient to say ahhh, with or without a nasty tongue depressor.

Paauw_Doug_SEATTLE_2019_web2.jpg
Dr. Douglas S. Paauw

When I was on my pediatrics rotation, I was taught to ask the patients to roar like a lion, which always gave a nice look at their posterior pharynx. The kids also really liked doing this, but it might seem a little strange to ask adults to do this.

A technique I have found that works well with adults is to ask them to yawn. I have found that this get me a great look at the pharynx for about half of my patients.
 

Auscultatory percussion for pleural effusions

Guarino and colleagues described a technique that is easily mastered and very effective for determining the presence of pleural effusions.1 It involves placing the stethoscope 3 cm below the last rib in the mid clavicular line and tapping from the apex down to the last rib.

For patients without effusion, a sharp change to a loud percussion note will occur at the last rib.

If the patient has an effusion, the loud percussion note will start at the top of the effusion.

This method was remarkably successful at finding pleural effusions. In the study, Dr. Guarino found a sensitivity of 96% and a specificity of 100%.
 

Physical exam for anemia

Look at the nails and see if they look pale. How can we do this?

The first step is to know what your own hematocrit is. You can then compare the color of your nail to that of the patient.

If you have a normal hematocrit and the patient’s nail bed color is lighter than yours, the patient likely has anemia. If you do this frequently, you will get good at estimating hematocrit. This is especially important if you do not have labs readily available.

Another way to assess for anemia is to look at the color tint of the lower conjunctiva. The best way to look for this is to look at whether there is a generous amount of visible capillaries in the lower conjunctiva. Patients without anemia have a darker red color because of these vessels, whereas patients with anemia are a lighter pink.

Strobach and colleagues2 looked at both nail bed rubor and color tint of the lower conjunctiva and found that both reliably predicted presence and degree of anemia.

 

 

Determining if clubbing is present

Most physicians are aware of Shamroth sign, and use it to evaluate for clubbing. Shamroth sign is the loss of the diamond that is created by placing the back surfaces of opposite terminal phalanges together.

I have found that it’s easier to diagnose mild clubbing by looking at the finger in profile. If the ratio of the distal phalangeal depth compared to the depth across the distal interphalangeal joint is greater than 1:1, then clubbing is present.3

Pearls

1. Have the patient try yawning to better see the pharynx without using a tongue blade.

2. Try the technique of auscultatory percussion to be more accurate at picking up pleural effusions.

3. Know your hematocrit, so you can better use color shade to assess for anemia.

4. Try looking at fingers in profile to pick up clubbing.

Dr. Paauw is professor of medicine in the division of general internal medicine at the University of Washington, Seattle, and serves as 3rd-year medical student clerkship director at the University of Washington. He is a member of the editorial advisory board of Internal Medicine News. Dr. Paauw has no conflicts to disclose. Contact him at imnews@mdedge.com.

References

1. Guarino JR and Guarino JC. Auscultatory percussion: A simple method to detect pleural effusion. J Gen Intern Med. 1994 Feb;9(2):71-4.

2. Strobach RS et al. The value of the physical examination in the diagnosis of anemia. Correlation of the physical findings and the hemoglobin concentration. Arch Intern Med. 1988 Apr;148(4):831-2.

3. Spicknall KE et al. Clubbing: an update on diagnosis, differential diagnosis, pathophysiology, and clinical relevance. J Am Acad Dermatol. 2005 Jun;52(6):1020-8.

 

I would like to start the new year off by returning to the past – when the physical exam was emphasized and utilized in decision making. I think a big reason that its use has diminished in recent years is due to the physical exam not having been emphasized in training.

For those seeking to increase their comfort with conducting the physical exam, below are several methods I have found helpful to use in practice.
 

Examining the pharynx

We were usually taught to ask the patient to say ahhh, with or without a nasty tongue depressor.

Paauw_Doug_SEATTLE_2019_web2.jpg
Dr. Douglas S. Paauw

When I was on my pediatrics rotation, I was taught to ask the patients to roar like a lion, which always gave a nice look at their posterior pharynx. The kids also really liked doing this, but it might seem a little strange to ask adults to do this.

A technique I have found that works well with adults is to ask them to yawn. I have found that this get me a great look at the pharynx for about half of my patients.
 

Auscultatory percussion for pleural effusions

Guarino and colleagues described a technique that is easily mastered and very effective for determining the presence of pleural effusions.1 It involves placing the stethoscope 3 cm below the last rib in the mid clavicular line and tapping from the apex down to the last rib.

For patients without effusion, a sharp change to a loud percussion note will occur at the last rib.

If the patient has an effusion, the loud percussion note will start at the top of the effusion.

This method was remarkably successful at finding pleural effusions. In the study, Dr. Guarino found a sensitivity of 96% and a specificity of 100%.
 

Physical exam for anemia

Look at the nails and see if they look pale. How can we do this?

The first step is to know what your own hematocrit is. You can then compare the color of your nail to that of the patient.

If you have a normal hematocrit and the patient’s nail bed color is lighter than yours, the patient likely has anemia. If you do this frequently, you will get good at estimating hematocrit. This is especially important if you do not have labs readily available.

Another way to assess for anemia is to look at the color tint of the lower conjunctiva. The best way to look for this is to look at whether there is a generous amount of visible capillaries in the lower conjunctiva. Patients without anemia have a darker red color because of these vessels, whereas patients with anemia are a lighter pink.

Strobach and colleagues2 looked at both nail bed rubor and color tint of the lower conjunctiva and found that both reliably predicted presence and degree of anemia.

 

 

Determining if clubbing is present

Most physicians are aware of Shamroth sign, and use it to evaluate for clubbing. Shamroth sign is the loss of the diamond that is created by placing the back surfaces of opposite terminal phalanges together.

I have found that it’s easier to diagnose mild clubbing by looking at the finger in profile. If the ratio of the distal phalangeal depth compared to the depth across the distal interphalangeal joint is greater than 1:1, then clubbing is present.3

Pearls

1. Have the patient try yawning to better see the pharynx without using a tongue blade.

2. Try the technique of auscultatory percussion to be more accurate at picking up pleural effusions.

3. Know your hematocrit, so you can better use color shade to assess for anemia.

4. Try looking at fingers in profile to pick up clubbing.

Dr. Paauw is professor of medicine in the division of general internal medicine at the University of Washington, Seattle, and serves as 3rd-year medical student clerkship director at the University of Washington. He is a member of the editorial advisory board of Internal Medicine News. Dr. Paauw has no conflicts to disclose. Contact him at imnews@mdedge.com.

References

1. Guarino JR and Guarino JC. Auscultatory percussion: A simple method to detect pleural effusion. J Gen Intern Med. 1994 Feb;9(2):71-4.

2. Strobach RS et al. The value of the physical examination in the diagnosis of anemia. Correlation of the physical findings and the hemoglobin concentration. Arch Intern Med. 1988 Apr;148(4):831-2.

3. Spicknall KE et al. Clubbing: an update on diagnosis, differential diagnosis, pathophysiology, and clinical relevance. J Am Acad Dermatol. 2005 Jun;52(6):1020-8.

Publications
Publications
Topics
Article Type
Sections
Disallow All Ads
Content Gating
No Gating (article Unlocked/Free)
Alternative CME
Disqus Comments
Default
Use ProPublica
Hide sidebar & use full width
render the right sidebar.
Conference Recap Checkbox
Not Conference Recap
Clinical Edge
Display the Slideshow in this Article
Medscape Article
Display survey writer
Reuters content
Disable Inline Native ads
WebMD Article